Suicide assessment and management self-test: How do you score?

Article Type
Changed
Thu, 03/28/2019 - 15:37
Display Headline
Suicide assessment and management self-test: How do you score?

As explained in the first part of this article in the October 2014 issue of Current Psychiatry, assess­ing and managing suicide risk are complex, difficult tasks without clear-cut, easy solutions. The case-based, mul­tiple-choice self-test, with accompanying commentary, pre­sented here is designed to enhance one’s ability to provide care for patients at risk for suicide. Part 2 of this article poses the remaining 7 of 15 questions, which are based on clinical experience and the referenced work of others.

Question 9
Mr. N, age 62, will be discharged from the psychiatric unit tomor­row. He was admitted after an overdose suicide attempt. Mr. N was depressed after the loss of his business and was “treating” his depres­sion and anxiety with alcohol. He is successfully withdrawn from alcohol and responds to medication and supportive psychotherapy. During a family meeting with staff, Mr. N’s wife states that he keeps a gun by his bedside. Mr. N has improved and is eager to go home.


Before discharging Mr. N, the psychiatrist or staff should:

   a) instruct Mr. N to remove the gun from his bedside
   b) instruct his wife to remove the gun from the home
   c) instruct the wife to look for >1 gun
   d) instruct the wife, before Mr. N’s discharge, to call the staff once guns and
ammunition are safely removed according to the pre-arranged safety plan
   e) instruct the wife to lock up the gun in a place that is not known to the patient


The best response option is D

Guns in the home are associated with a sig­nificant increase in suicide. All patients at risk for suicide must be asked if guns are available at home or easily accessible else­where, or if they intend to purchase a gun. Gun safety management requires a collabor­ative team approach including the clinician, patient, and person designated responsible for removing guns from the home.1 The responsible person should be required to call the clinician to confirm that the guns have been removed and secured according to the plan. The principles of gun safety manage­ment apply to outpatients, inpatients, and emergency patients, although implementa­tion varies according to the clinical setting.

Asking the patient to remove guns from the home is too risky. Guns must be safely secured before the patient is discharged. Asking a spouse, other family member, or partner is necessary. The person asked must be willing to remove guns and ammunition according to a pre-arranged plan requiring a callback upon completion. A callback is essential because a family member in denial may do nothing to remove the guns or lock or “hide” them in the home where they will be found by a determined suicidal patient. Guns may be available outside the home, such as in the car, at the work place, or for purchase.

The essence of gun safety management is verification. Trust but verify or, better yet, verify, then trust.

Question 10
A recently admitted 56-year-old inpatient was discovered wrapping a towel around her neck. She denied suicidal intent; however, the treat­ment team viewed the incident as a suicide rehearsal. She was placed on one-to-one close observation.

Inpatient suicides frequently occur:
   a) shortly after admission
   b) during staff shift changes
   c) at meal times
   d) shortly after discharge
   e) all of the above

The best response option is E
Inpatient suicides also occur at increased frequency when psychiatric residents fin­ish their rotations and in understaffed psychiatric units.2 Undue delay in the evaluation of a newly admitted acute, high-risk patient might allow the patient to commit suicide.

Most patient suicides occur shortly after hospital discharge (a few hours, days, or weeks later). Appleby et al3 found that the highest number of suicides occurred during the first week after discharge. Meehan et al4 found that suicide occurred most frequently during the first 2 weeks post-discharge; the highest number of suicides occurred on the first day after discharge.

Question 11
Ms. G, a 43-year-old, single woman in acute sui­cide crisis, is admitted to the psychiatric unit of a general hospital. She is diagnosed with bipolar I disorder, most recent episode depressed, and borderline personality disorder. She has had multiple psychiatric hospitalizations, all pre­cipitated by a suicide crisis. The average length of stay on the psychiatric unit is 6.3 days. After 7 days of intensive treatment, Ms. G is stabilized and suicide risk is reduced. The treatment team prepares for her discharge.


Ms. G’s suicide risk at discharge is most likely at:

   a) indeterminate risk
   b) low risk
   c) moderate risk
   d) chronic high risk
   e) acute high risk

 

 


The best response option is D
The length of stay in many acute care psy­chiatric facilities is <7 days. The goal of hospitalization is to stabilize the patient and discharge to appropriate community mental health resources. Discharge plan­ning begins at the time of admission.

Reducing Ms. G’s suicide risk to low or moderate is unlikely because of her diagnoses, frequent hospitalizations, and acute high risk for suicide on admission. After acute, high-risk suicidal patients are treated, many revert to chronic high risk for suicide.

Patients at chronic high risk for suicide often are treated as outpatients, except when an acute suicidal crisis requires hospi­talization.5 At discharge from the hospital, the goal is to return the patient to outpatient treatment.

A discharge note identifies the acute sui­cide risk factors that have abated and the chronic (long-term) suicide risk factors that remain. The discharge note also addresses a patient’s chronic vulnerability to suicide. For example, a patient can become acutely suicidal again, depending on a number of factors, including the nature and cause of the psychiatric illness, adequacy of future treat­ment, adherence to treatment recommenda­tions, and unforeseeable life vicissitudes.

Question 12
A 20-year-old college student is hospitalized after an overdose suicide attempt. Failing grades, panic attacks, and depression precipitated the suicide attempt. After 8 days of hos­pitalization, she is much improved and ready for discharge. She is assessed to be at low to moderate suicide risk. The treating psychiatrist and social worker convene a family meet­ing with both parents and an older brother. The family’s role after discharge is discussed.


All of the following options are helpful family roles except:

   a) provide constant 24-hour family supervision
   b) provide emotional support
   c) observe and report symptoms and behaviors of concern
   d) encourage adherence with treatment
   e) provide helpful feedback about the patient’s thoughts and behavior


The best response option is A
The family’s role is important, but it is not a substitute for constant safety management provided by trained mental health profes­sionals on an inpatient psychiatric unit.5 Early discharge of an inpatient by relying on family supervision can be precarious. Most inpatients are discharged at some level of suicide risk, given the short length of hos­pital stay. If an outpatient at risk of suicide requires constant 24-hour family supervi­sion, then psychiatric hospitalization is indicated.

Patients who are intent on killing them­selves can find ingenious ways to attempt or commit suicide. Asking family members to keep a constant watch often fails. Most family members will not follow the patient into the bathroom or be able to stay up all night to observe the patient. Moreover, fam­ily members find reasons to make exceptions to constant surveillance because of denial, fatigue, or the need to attend to other press­ing matters.


Question 13

During the initial evaluation of a patient, it is the psychiatrist’s practice to routinely inquire about current and past suicide ideation. An affirma­tive answer prompts a systematic suicide risk assessment. In the absence of current risk, if exploration of the patient’s history reveals chronic suicide risk factors, the psychiatrist con­ducts a systematic suicide risk assessment.


The chronic risk factor that has the highest association with suicide is:

   a) family history of mental illness or suicide
   b) childhood abuse
   c) history of a suicide attempt
   d) impulsivity or aggression
   e) prior psychiatric hospitalization


The best response option is C
A comprehensive suicide risk assessment may not be required at the initial outpatient evaluation in the absence of acute suicide risk factors. However, chronic suicide risk factors may be present.

The Standard Mortality Ratio (SMR) for prior suicide attempts by any method was 38.61.6 Suicide risk was highest in the 2 years after the first attempt. The SMR is a measure of the relative risk of suicide compared with the expected rate in the general population (SMR of 1).

Some chronic suicide risk factors are static: for example, a family history of psy­chiatric illness or earlier suicide attempt. Other chronic risk factors, usually a trait characteristic, can become acute: for exam­ple, impulsivity or aggression, or deliberate self-harm. The presence of chronic suicide risk factors should prompt a systematic sui­cide risk assessment. Evaluation of chronic suicide risk factors is an essential component of comprehensive assessment.5


Question 14

A psychiatrist is treating Dr. R, a 43-year-old physician, for anxiety and depression. The psychiatrist sees Dr. R twice a week for psy­chotherapy and medication management. A recent lawsuit filed against Dr. R has severely exacerbated her symptoms. She can sleep for only a few hours. Suicide ideation has emerged, frightening Dr. R and her family. The psychiatrist performs a systematic suicide risk assessment and determines that Dr. R is at acute high risk for suicide.

The psychiatrist recommends immediate hospitalization, but Dr. R adamantly refuses. The psychiatrist decides not to involuntarily hospitalize her because she does not meet the substantive criteria of the state involun­tary commitment statute (eg, overt suicidal behaviors). The psychiatrist chooses to con­tinue outpatient treatment.

 

 


Clinical interventions to reduce Dr. R’s suicide risk include:

   a) see her more often
   b) adjust medications
   c) obtain a consult
   d) refer her to an intensive outpatient program
   e) all of the above


The best response option is E

To hospitalize or not to hospitalize— that is the conundrum that psychiatrists often face with high-risk suicidal patients. The decision is more complicated when the need for hospitalization is clear but the patient refuses. The decisions that the psychiatrist makes at this point are crucial for treatment and risk management.5

If the patient disagrees with the psychiatrist’s recommendation to hospitalize, refusal should be addressed as a treatment issue. When the need for hospitalization is acute, a prolonged inquiry is not possible. In addition, the therapeutic alliance may become strained. This clinical situation tries a clinician’s professional mettle.

Consultation and referral are options to consider if time and the patient’s condition allows. A psychiatric clinician should never worry alone; sleepless nights benefit neither the psychiatrist nor the patient.

As Dr. R’s case shows, a psychiatrist might decide not to hospitalize a patient who is assessed to be at moderate or high risk of suicide. Protective factors may allow continuing outpatient treatment. A good therapeutic alliance may be present if the psychiatrist has worked with the patient for some time. Family support also may be available.

The clinician must determine if the patient’s suicide risk can be managed by more frequent visits and treatment adjust­ments. Also, supportive family members can help by providing observational data. Protective factors can be overwhelmed by a severe mental illness. In contrast, a patient assessed as being at moderate risk of suicide might need to be hospitalized when protec­tive factors are few or absent.

The psychiatrist may determine that a patient at high risk of suicide who refuses hospitalization does not meet criteria for involuntary hospitalization. For example, criteria might require that the patient must have made a suicide attempt within a speci­fied period of time. States have provisions in their commitment statutes granting immu­nity from liability if the clinician uses reason­able clinical judgment and acts in good faith when involuntarily hospitalizing a patient.7 

Question 15
Mr. U, a 39-year-old, married engineer, is ready to be discharged from the inpatient unit. He was admitted 7 days earlier for acute alco­hol intoxication and suicidal threats. He has undergone successful detoxification. Mr. U has had 2 similar episodes within the past year.

The treatment team conducts a risk-benefit analysis for both discharge and continued hospi­talization. A consultation also is obtained.


The discharge decision will be most influenced by:

   a) presence of family support
   b) compliance with follow-up care
   c) availability of dual diagnosis programs
   d) systematic suicide risk assessment
   e) consultation


The best response option is D

All of the options in Question 15 concerning discharge planning of patients at risk for sui­cide are important. However, conducting a systematic suicide risk assessment to inform discharge planning is the most critical. Mr. U had 2 previous psychiatric admissions for alcohol abuse and suicidal ideation. He is a chronic suicide risk who becomes high risk when intoxicated.

Discharge planning begins at admission and is refined during the patient’s stay. Before a patient is discharged, a final post-discharge treatment and aftercare plan is necessary. After discharge, suicide risk increases as the intensity of treatment decreases.8

The patient’s willingness to cooperate with discharge and aftercare planning is critical in establishing contact with follow-up treat­ers. The treatment team should structure the follow-up plan to encourage compliance. For example, psychotic patients at risk of suicide who have a history of stopping medications after discharge can be given a long-acting IM antipsychotic that will last until they reach aftercare. Patients with comorbid drug and alcohol abuse disorders are referred to agen­cies equipped to manage dual-diagnosis patients.

Psychiatrists’ ability to ensure follow-up treatment is limited, a fact that must be acknowledged by the psychiatric and legal communities. Beyond patient stabilization, a clinician’s options to bring about posi­tive changes can be limited or nonexistent. Also, the patient’s failure to adhere to post-discharge plans and treatment often leads to rehospitalization, hopelessness, and greater suicide risk.

Psychiatric patients at moderate or moderate-to-high risk for suicide increas­ingly are treated in outpatient settings. It is the responsibility of the clinician and the treatment team to competently hand off the patient to appropriate outpatient aftercare. With the patient’s permission, the psychia­trist or social worker should call the follow-up agency or therapist before discharge to provide information about the patient’s diagnosis, treatment, and hospital course.

Last, follow-up appointments should be made as close to the time of discharge as possible. Suicide often occurs on the first day after discharge.3

 

 

Bottom Line
Fully commit time and effort to the ongoing assessment, treatment, and management of patients at suicide risk. Suicide risk assessment is a process, not an event. Conduct a suicide risk assessment at important clinical junctures (eg, initial evaluation, discharge, changing observation levels). Contemporaneously, document suicide risk assessments. This self-assessment helps clinicians gauge their strengths and identify skills that need further development.

Disclosure
Dr. Simon reports no financial relationship with any company whose products are mentioned in this article or with manufacturers of competing products.

Dr. Simon is the co-editor of The American Psychiatry Publishing textbook of Suicide Assessment and Management, 2nd edition, from which this article is adapted, by permission of the publisher, American Psychiatry Publishing, Inc. ©2012.

References


1. Simon RI. Gun safety management with patients at risk for suicide. Suicide Life Threat Behav. 2007;37(5):518-526.
2. Qin P, Nordenoft M. Suicide risk in relation to psychiatric hospitalization: evidence based on longitudinal registers. Arch Gen Psychiatry. 2005;62(4):427-432.
3. Appleby L, Shaw J, Amos T, et al. Suicide within 12 months of contact with mental health services: national clinical survey. BMJ. 1999;318(7193):1235-1239.
4. Meehan J, Kapur N, Hunt IM, et al. Suicide in mental health in-patients within 3 months of discharge. National clinical survey. Br J Psychiatry. 2006;188:129-134.
5. Simon RI. Preventing patient suicide: clinical assessment and management. Arlington, VA: American Psychiatric Publishing, Inc.; 2011.
6. Harris CE, Barraclough B. Suicide as an outcome for mental disorders. A meta-analysis. Br J Psychiatry. 1997;170:205-228.
7. Simon RI, Shuman DW. Clinical manual of psychiatry and law. Arlington, VA: American Psychiatric Publishing, Inc.; 2007.
8. Appleby L, Dennehy JA, Thomas CS, et al. Aftercare and clinical characteristics of people with mental illness who commit suicide: a case-control study. Lancet. 1999;353(9162):1397-1400.

Article PDF
Author and Disclosure Information

Robert I. Simon, MD
Clinical Professor of Psychiatry
Georgetown University School of Medicine
Washington, DC

Issue
Current Psychiatry - 13(11)
Publications
Topics
Page Number
21-22, 24-25, 32
Legacy Keywords
suicide risk assessment, suicide, depression, guns, involuntary hospitalization, hospitalization
Sections
Author and Disclosure Information

Robert I. Simon, MD
Clinical Professor of Psychiatry
Georgetown University School of Medicine
Washington, DC

Author and Disclosure Information

Robert I. Simon, MD
Clinical Professor of Psychiatry
Georgetown University School of Medicine
Washington, DC

Article PDF
Article PDF
Related Articles

As explained in the first part of this article in the October 2014 issue of Current Psychiatry, assess­ing and managing suicide risk are complex, difficult tasks without clear-cut, easy solutions. The case-based, mul­tiple-choice self-test, with accompanying commentary, pre­sented here is designed to enhance one’s ability to provide care for patients at risk for suicide. Part 2 of this article poses the remaining 7 of 15 questions, which are based on clinical experience and the referenced work of others.

Question 9
Mr. N, age 62, will be discharged from the psychiatric unit tomor­row. He was admitted after an overdose suicide attempt. Mr. N was depressed after the loss of his business and was “treating” his depres­sion and anxiety with alcohol. He is successfully withdrawn from alcohol and responds to medication and supportive psychotherapy. During a family meeting with staff, Mr. N’s wife states that he keeps a gun by his bedside. Mr. N has improved and is eager to go home.


Before discharging Mr. N, the psychiatrist or staff should:

   a) instruct Mr. N to remove the gun from his bedside
   b) instruct his wife to remove the gun from the home
   c) instruct the wife to look for >1 gun
   d) instruct the wife, before Mr. N’s discharge, to call the staff once guns and
ammunition are safely removed according to the pre-arranged safety plan
   e) instruct the wife to lock up the gun in a place that is not known to the patient


The best response option is D

Guns in the home are associated with a sig­nificant increase in suicide. All patients at risk for suicide must be asked if guns are available at home or easily accessible else­where, or if they intend to purchase a gun. Gun safety management requires a collabor­ative team approach including the clinician, patient, and person designated responsible for removing guns from the home.1 The responsible person should be required to call the clinician to confirm that the guns have been removed and secured according to the plan. The principles of gun safety manage­ment apply to outpatients, inpatients, and emergency patients, although implementa­tion varies according to the clinical setting.

Asking the patient to remove guns from the home is too risky. Guns must be safely secured before the patient is discharged. Asking a spouse, other family member, or partner is necessary. The person asked must be willing to remove guns and ammunition according to a pre-arranged plan requiring a callback upon completion. A callback is essential because a family member in denial may do nothing to remove the guns or lock or “hide” them in the home where they will be found by a determined suicidal patient. Guns may be available outside the home, such as in the car, at the work place, or for purchase.

The essence of gun safety management is verification. Trust but verify or, better yet, verify, then trust.

Question 10
A recently admitted 56-year-old inpatient was discovered wrapping a towel around her neck. She denied suicidal intent; however, the treat­ment team viewed the incident as a suicide rehearsal. She was placed on one-to-one close observation.

Inpatient suicides frequently occur:
   a) shortly after admission
   b) during staff shift changes
   c) at meal times
   d) shortly after discharge
   e) all of the above

The best response option is E
Inpatient suicides also occur at increased frequency when psychiatric residents fin­ish their rotations and in understaffed psychiatric units.2 Undue delay in the evaluation of a newly admitted acute, high-risk patient might allow the patient to commit suicide.

Most patient suicides occur shortly after hospital discharge (a few hours, days, or weeks later). Appleby et al3 found that the highest number of suicides occurred during the first week after discharge. Meehan et al4 found that suicide occurred most frequently during the first 2 weeks post-discharge; the highest number of suicides occurred on the first day after discharge.

Question 11
Ms. G, a 43-year-old, single woman in acute sui­cide crisis, is admitted to the psychiatric unit of a general hospital. She is diagnosed with bipolar I disorder, most recent episode depressed, and borderline personality disorder. She has had multiple psychiatric hospitalizations, all pre­cipitated by a suicide crisis. The average length of stay on the psychiatric unit is 6.3 days. After 7 days of intensive treatment, Ms. G is stabilized and suicide risk is reduced. The treatment team prepares for her discharge.


Ms. G’s suicide risk at discharge is most likely at:

   a) indeterminate risk
   b) low risk
   c) moderate risk
   d) chronic high risk
   e) acute high risk

 

 


The best response option is D
The length of stay in many acute care psy­chiatric facilities is <7 days. The goal of hospitalization is to stabilize the patient and discharge to appropriate community mental health resources. Discharge plan­ning begins at the time of admission.

Reducing Ms. G’s suicide risk to low or moderate is unlikely because of her diagnoses, frequent hospitalizations, and acute high risk for suicide on admission. After acute, high-risk suicidal patients are treated, many revert to chronic high risk for suicide.

Patients at chronic high risk for suicide often are treated as outpatients, except when an acute suicidal crisis requires hospi­talization.5 At discharge from the hospital, the goal is to return the patient to outpatient treatment.

A discharge note identifies the acute sui­cide risk factors that have abated and the chronic (long-term) suicide risk factors that remain. The discharge note also addresses a patient’s chronic vulnerability to suicide. For example, a patient can become acutely suicidal again, depending on a number of factors, including the nature and cause of the psychiatric illness, adequacy of future treat­ment, adherence to treatment recommenda­tions, and unforeseeable life vicissitudes.

Question 12
A 20-year-old college student is hospitalized after an overdose suicide attempt. Failing grades, panic attacks, and depression precipitated the suicide attempt. After 8 days of hos­pitalization, she is much improved and ready for discharge. She is assessed to be at low to moderate suicide risk. The treating psychiatrist and social worker convene a family meet­ing with both parents and an older brother. The family’s role after discharge is discussed.


All of the following options are helpful family roles except:

   a) provide constant 24-hour family supervision
   b) provide emotional support
   c) observe and report symptoms and behaviors of concern
   d) encourage adherence with treatment
   e) provide helpful feedback about the patient’s thoughts and behavior


The best response option is A
The family’s role is important, but it is not a substitute for constant safety management provided by trained mental health profes­sionals on an inpatient psychiatric unit.5 Early discharge of an inpatient by relying on family supervision can be precarious. Most inpatients are discharged at some level of suicide risk, given the short length of hos­pital stay. If an outpatient at risk of suicide requires constant 24-hour family supervi­sion, then psychiatric hospitalization is indicated.

Patients who are intent on killing them­selves can find ingenious ways to attempt or commit suicide. Asking family members to keep a constant watch often fails. Most family members will not follow the patient into the bathroom or be able to stay up all night to observe the patient. Moreover, fam­ily members find reasons to make exceptions to constant surveillance because of denial, fatigue, or the need to attend to other press­ing matters.


Question 13

During the initial evaluation of a patient, it is the psychiatrist’s practice to routinely inquire about current and past suicide ideation. An affirma­tive answer prompts a systematic suicide risk assessment. In the absence of current risk, if exploration of the patient’s history reveals chronic suicide risk factors, the psychiatrist con­ducts a systematic suicide risk assessment.


The chronic risk factor that has the highest association with suicide is:

   a) family history of mental illness or suicide
   b) childhood abuse
   c) history of a suicide attempt
   d) impulsivity or aggression
   e) prior psychiatric hospitalization


The best response option is C
A comprehensive suicide risk assessment may not be required at the initial outpatient evaluation in the absence of acute suicide risk factors. However, chronic suicide risk factors may be present.

The Standard Mortality Ratio (SMR) for prior suicide attempts by any method was 38.61.6 Suicide risk was highest in the 2 years after the first attempt. The SMR is a measure of the relative risk of suicide compared with the expected rate in the general population (SMR of 1).

Some chronic suicide risk factors are static: for example, a family history of psy­chiatric illness or earlier suicide attempt. Other chronic risk factors, usually a trait characteristic, can become acute: for exam­ple, impulsivity or aggression, or deliberate self-harm. The presence of chronic suicide risk factors should prompt a systematic sui­cide risk assessment. Evaluation of chronic suicide risk factors is an essential component of comprehensive assessment.5


Question 14

A psychiatrist is treating Dr. R, a 43-year-old physician, for anxiety and depression. The psychiatrist sees Dr. R twice a week for psy­chotherapy and medication management. A recent lawsuit filed against Dr. R has severely exacerbated her symptoms. She can sleep for only a few hours. Suicide ideation has emerged, frightening Dr. R and her family. The psychiatrist performs a systematic suicide risk assessment and determines that Dr. R is at acute high risk for suicide.

The psychiatrist recommends immediate hospitalization, but Dr. R adamantly refuses. The psychiatrist decides not to involuntarily hospitalize her because she does not meet the substantive criteria of the state involun­tary commitment statute (eg, overt suicidal behaviors). The psychiatrist chooses to con­tinue outpatient treatment.

 

 


Clinical interventions to reduce Dr. R’s suicide risk include:

   a) see her more often
   b) adjust medications
   c) obtain a consult
   d) refer her to an intensive outpatient program
   e) all of the above


The best response option is E

To hospitalize or not to hospitalize— that is the conundrum that psychiatrists often face with high-risk suicidal patients. The decision is more complicated when the need for hospitalization is clear but the patient refuses. The decisions that the psychiatrist makes at this point are crucial for treatment and risk management.5

If the patient disagrees with the psychiatrist’s recommendation to hospitalize, refusal should be addressed as a treatment issue. When the need for hospitalization is acute, a prolonged inquiry is not possible. In addition, the therapeutic alliance may become strained. This clinical situation tries a clinician’s professional mettle.

Consultation and referral are options to consider if time and the patient’s condition allows. A psychiatric clinician should never worry alone; sleepless nights benefit neither the psychiatrist nor the patient.

As Dr. R’s case shows, a psychiatrist might decide not to hospitalize a patient who is assessed to be at moderate or high risk of suicide. Protective factors may allow continuing outpatient treatment. A good therapeutic alliance may be present if the psychiatrist has worked with the patient for some time. Family support also may be available.

The clinician must determine if the patient’s suicide risk can be managed by more frequent visits and treatment adjust­ments. Also, supportive family members can help by providing observational data. Protective factors can be overwhelmed by a severe mental illness. In contrast, a patient assessed as being at moderate risk of suicide might need to be hospitalized when protec­tive factors are few or absent.

The psychiatrist may determine that a patient at high risk of suicide who refuses hospitalization does not meet criteria for involuntary hospitalization. For example, criteria might require that the patient must have made a suicide attempt within a speci­fied period of time. States have provisions in their commitment statutes granting immu­nity from liability if the clinician uses reason­able clinical judgment and acts in good faith when involuntarily hospitalizing a patient.7 

Question 15
Mr. U, a 39-year-old, married engineer, is ready to be discharged from the inpatient unit. He was admitted 7 days earlier for acute alco­hol intoxication and suicidal threats. He has undergone successful detoxification. Mr. U has had 2 similar episodes within the past year.

The treatment team conducts a risk-benefit analysis for both discharge and continued hospi­talization. A consultation also is obtained.


The discharge decision will be most influenced by:

   a) presence of family support
   b) compliance with follow-up care
   c) availability of dual diagnosis programs
   d) systematic suicide risk assessment
   e) consultation


The best response option is D

All of the options in Question 15 concerning discharge planning of patients at risk for sui­cide are important. However, conducting a systematic suicide risk assessment to inform discharge planning is the most critical. Mr. U had 2 previous psychiatric admissions for alcohol abuse and suicidal ideation. He is a chronic suicide risk who becomes high risk when intoxicated.

Discharge planning begins at admission and is refined during the patient’s stay. Before a patient is discharged, a final post-discharge treatment and aftercare plan is necessary. After discharge, suicide risk increases as the intensity of treatment decreases.8

The patient’s willingness to cooperate with discharge and aftercare planning is critical in establishing contact with follow-up treat­ers. The treatment team should structure the follow-up plan to encourage compliance. For example, psychotic patients at risk of suicide who have a history of stopping medications after discharge can be given a long-acting IM antipsychotic that will last until they reach aftercare. Patients with comorbid drug and alcohol abuse disorders are referred to agen­cies equipped to manage dual-diagnosis patients.

Psychiatrists’ ability to ensure follow-up treatment is limited, a fact that must be acknowledged by the psychiatric and legal communities. Beyond patient stabilization, a clinician’s options to bring about posi­tive changes can be limited or nonexistent. Also, the patient’s failure to adhere to post-discharge plans and treatment often leads to rehospitalization, hopelessness, and greater suicide risk.

Psychiatric patients at moderate or moderate-to-high risk for suicide increas­ingly are treated in outpatient settings. It is the responsibility of the clinician and the treatment team to competently hand off the patient to appropriate outpatient aftercare. With the patient’s permission, the psychia­trist or social worker should call the follow-up agency or therapist before discharge to provide information about the patient’s diagnosis, treatment, and hospital course.

Last, follow-up appointments should be made as close to the time of discharge as possible. Suicide often occurs on the first day after discharge.3

 

 

Bottom Line
Fully commit time and effort to the ongoing assessment, treatment, and management of patients at suicide risk. Suicide risk assessment is a process, not an event. Conduct a suicide risk assessment at important clinical junctures (eg, initial evaluation, discharge, changing observation levels). Contemporaneously, document suicide risk assessments. This self-assessment helps clinicians gauge their strengths and identify skills that need further development.

Disclosure
Dr. Simon reports no financial relationship with any company whose products are mentioned in this article or with manufacturers of competing products.

Dr. Simon is the co-editor of The American Psychiatry Publishing textbook of Suicide Assessment and Management, 2nd edition, from which this article is adapted, by permission of the publisher, American Psychiatry Publishing, Inc. ©2012.

As explained in the first part of this article in the October 2014 issue of Current Psychiatry, assess­ing and managing suicide risk are complex, difficult tasks without clear-cut, easy solutions. The case-based, mul­tiple-choice self-test, with accompanying commentary, pre­sented here is designed to enhance one’s ability to provide care for patients at risk for suicide. Part 2 of this article poses the remaining 7 of 15 questions, which are based on clinical experience and the referenced work of others.

Question 9
Mr. N, age 62, will be discharged from the psychiatric unit tomor­row. He was admitted after an overdose suicide attempt. Mr. N was depressed after the loss of his business and was “treating” his depres­sion and anxiety with alcohol. He is successfully withdrawn from alcohol and responds to medication and supportive psychotherapy. During a family meeting with staff, Mr. N’s wife states that he keeps a gun by his bedside. Mr. N has improved and is eager to go home.


Before discharging Mr. N, the psychiatrist or staff should:

   a) instruct Mr. N to remove the gun from his bedside
   b) instruct his wife to remove the gun from the home
   c) instruct the wife to look for >1 gun
   d) instruct the wife, before Mr. N’s discharge, to call the staff once guns and
ammunition are safely removed according to the pre-arranged safety plan
   e) instruct the wife to lock up the gun in a place that is not known to the patient


The best response option is D

Guns in the home are associated with a sig­nificant increase in suicide. All patients at risk for suicide must be asked if guns are available at home or easily accessible else­where, or if they intend to purchase a gun. Gun safety management requires a collabor­ative team approach including the clinician, patient, and person designated responsible for removing guns from the home.1 The responsible person should be required to call the clinician to confirm that the guns have been removed and secured according to the plan. The principles of gun safety manage­ment apply to outpatients, inpatients, and emergency patients, although implementa­tion varies according to the clinical setting.

Asking the patient to remove guns from the home is too risky. Guns must be safely secured before the patient is discharged. Asking a spouse, other family member, or partner is necessary. The person asked must be willing to remove guns and ammunition according to a pre-arranged plan requiring a callback upon completion. A callback is essential because a family member in denial may do nothing to remove the guns or lock or “hide” them in the home where they will be found by a determined suicidal patient. Guns may be available outside the home, such as in the car, at the work place, or for purchase.

The essence of gun safety management is verification. Trust but verify or, better yet, verify, then trust.

Question 10
A recently admitted 56-year-old inpatient was discovered wrapping a towel around her neck. She denied suicidal intent; however, the treat­ment team viewed the incident as a suicide rehearsal. She was placed on one-to-one close observation.

Inpatient suicides frequently occur:
   a) shortly after admission
   b) during staff shift changes
   c) at meal times
   d) shortly after discharge
   e) all of the above

The best response option is E
Inpatient suicides also occur at increased frequency when psychiatric residents fin­ish their rotations and in understaffed psychiatric units.2 Undue delay in the evaluation of a newly admitted acute, high-risk patient might allow the patient to commit suicide.

Most patient suicides occur shortly after hospital discharge (a few hours, days, or weeks later). Appleby et al3 found that the highest number of suicides occurred during the first week after discharge. Meehan et al4 found that suicide occurred most frequently during the first 2 weeks post-discharge; the highest number of suicides occurred on the first day after discharge.

Question 11
Ms. G, a 43-year-old, single woman in acute sui­cide crisis, is admitted to the psychiatric unit of a general hospital. She is diagnosed with bipolar I disorder, most recent episode depressed, and borderline personality disorder. She has had multiple psychiatric hospitalizations, all pre­cipitated by a suicide crisis. The average length of stay on the psychiatric unit is 6.3 days. After 7 days of intensive treatment, Ms. G is stabilized and suicide risk is reduced. The treatment team prepares for her discharge.


Ms. G’s suicide risk at discharge is most likely at:

   a) indeterminate risk
   b) low risk
   c) moderate risk
   d) chronic high risk
   e) acute high risk

 

 


The best response option is D
The length of stay in many acute care psy­chiatric facilities is <7 days. The goal of hospitalization is to stabilize the patient and discharge to appropriate community mental health resources. Discharge plan­ning begins at the time of admission.

Reducing Ms. G’s suicide risk to low or moderate is unlikely because of her diagnoses, frequent hospitalizations, and acute high risk for suicide on admission. After acute, high-risk suicidal patients are treated, many revert to chronic high risk for suicide.

Patients at chronic high risk for suicide often are treated as outpatients, except when an acute suicidal crisis requires hospi­talization.5 At discharge from the hospital, the goal is to return the patient to outpatient treatment.

A discharge note identifies the acute sui­cide risk factors that have abated and the chronic (long-term) suicide risk factors that remain. The discharge note also addresses a patient’s chronic vulnerability to suicide. For example, a patient can become acutely suicidal again, depending on a number of factors, including the nature and cause of the psychiatric illness, adequacy of future treat­ment, adherence to treatment recommenda­tions, and unforeseeable life vicissitudes.

Question 12
A 20-year-old college student is hospitalized after an overdose suicide attempt. Failing grades, panic attacks, and depression precipitated the suicide attempt. After 8 days of hos­pitalization, she is much improved and ready for discharge. She is assessed to be at low to moderate suicide risk. The treating psychiatrist and social worker convene a family meet­ing with both parents and an older brother. The family’s role after discharge is discussed.


All of the following options are helpful family roles except:

   a) provide constant 24-hour family supervision
   b) provide emotional support
   c) observe and report symptoms and behaviors of concern
   d) encourage adherence with treatment
   e) provide helpful feedback about the patient’s thoughts and behavior


The best response option is A
The family’s role is important, but it is not a substitute for constant safety management provided by trained mental health profes­sionals on an inpatient psychiatric unit.5 Early discharge of an inpatient by relying on family supervision can be precarious. Most inpatients are discharged at some level of suicide risk, given the short length of hos­pital stay. If an outpatient at risk of suicide requires constant 24-hour family supervi­sion, then psychiatric hospitalization is indicated.

Patients who are intent on killing them­selves can find ingenious ways to attempt or commit suicide. Asking family members to keep a constant watch often fails. Most family members will not follow the patient into the bathroom or be able to stay up all night to observe the patient. Moreover, fam­ily members find reasons to make exceptions to constant surveillance because of denial, fatigue, or the need to attend to other press­ing matters.


Question 13

During the initial evaluation of a patient, it is the psychiatrist’s practice to routinely inquire about current and past suicide ideation. An affirma­tive answer prompts a systematic suicide risk assessment. In the absence of current risk, if exploration of the patient’s history reveals chronic suicide risk factors, the psychiatrist con­ducts a systematic suicide risk assessment.


The chronic risk factor that has the highest association with suicide is:

   a) family history of mental illness or suicide
   b) childhood abuse
   c) history of a suicide attempt
   d) impulsivity or aggression
   e) prior psychiatric hospitalization


The best response option is C
A comprehensive suicide risk assessment may not be required at the initial outpatient evaluation in the absence of acute suicide risk factors. However, chronic suicide risk factors may be present.

The Standard Mortality Ratio (SMR) for prior suicide attempts by any method was 38.61.6 Suicide risk was highest in the 2 years after the first attempt. The SMR is a measure of the relative risk of suicide compared with the expected rate in the general population (SMR of 1).

Some chronic suicide risk factors are static: for example, a family history of psy­chiatric illness or earlier suicide attempt. Other chronic risk factors, usually a trait characteristic, can become acute: for exam­ple, impulsivity or aggression, or deliberate self-harm. The presence of chronic suicide risk factors should prompt a systematic sui­cide risk assessment. Evaluation of chronic suicide risk factors is an essential component of comprehensive assessment.5


Question 14

A psychiatrist is treating Dr. R, a 43-year-old physician, for anxiety and depression. The psychiatrist sees Dr. R twice a week for psy­chotherapy and medication management. A recent lawsuit filed against Dr. R has severely exacerbated her symptoms. She can sleep for only a few hours. Suicide ideation has emerged, frightening Dr. R and her family. The psychiatrist performs a systematic suicide risk assessment and determines that Dr. R is at acute high risk for suicide.

The psychiatrist recommends immediate hospitalization, but Dr. R adamantly refuses. The psychiatrist decides not to involuntarily hospitalize her because she does not meet the substantive criteria of the state involun­tary commitment statute (eg, overt suicidal behaviors). The psychiatrist chooses to con­tinue outpatient treatment.

 

 


Clinical interventions to reduce Dr. R’s suicide risk include:

   a) see her more often
   b) adjust medications
   c) obtain a consult
   d) refer her to an intensive outpatient program
   e) all of the above


The best response option is E

To hospitalize or not to hospitalize— that is the conundrum that psychiatrists often face with high-risk suicidal patients. The decision is more complicated when the need for hospitalization is clear but the patient refuses. The decisions that the psychiatrist makes at this point are crucial for treatment and risk management.5

If the patient disagrees with the psychiatrist’s recommendation to hospitalize, refusal should be addressed as a treatment issue. When the need for hospitalization is acute, a prolonged inquiry is not possible. In addition, the therapeutic alliance may become strained. This clinical situation tries a clinician’s professional mettle.

Consultation and referral are options to consider if time and the patient’s condition allows. A psychiatric clinician should never worry alone; sleepless nights benefit neither the psychiatrist nor the patient.

As Dr. R’s case shows, a psychiatrist might decide not to hospitalize a patient who is assessed to be at moderate or high risk of suicide. Protective factors may allow continuing outpatient treatment. A good therapeutic alliance may be present if the psychiatrist has worked with the patient for some time. Family support also may be available.

The clinician must determine if the patient’s suicide risk can be managed by more frequent visits and treatment adjust­ments. Also, supportive family members can help by providing observational data. Protective factors can be overwhelmed by a severe mental illness. In contrast, a patient assessed as being at moderate risk of suicide might need to be hospitalized when protec­tive factors are few or absent.

The psychiatrist may determine that a patient at high risk of suicide who refuses hospitalization does not meet criteria for involuntary hospitalization. For example, criteria might require that the patient must have made a suicide attempt within a speci­fied period of time. States have provisions in their commitment statutes granting immu­nity from liability if the clinician uses reason­able clinical judgment and acts in good faith when involuntarily hospitalizing a patient.7 

Question 15
Mr. U, a 39-year-old, married engineer, is ready to be discharged from the inpatient unit. He was admitted 7 days earlier for acute alco­hol intoxication and suicidal threats. He has undergone successful detoxification. Mr. U has had 2 similar episodes within the past year.

The treatment team conducts a risk-benefit analysis for both discharge and continued hospi­talization. A consultation also is obtained.


The discharge decision will be most influenced by:

   a) presence of family support
   b) compliance with follow-up care
   c) availability of dual diagnosis programs
   d) systematic suicide risk assessment
   e) consultation


The best response option is D

All of the options in Question 15 concerning discharge planning of patients at risk for sui­cide are important. However, conducting a systematic suicide risk assessment to inform discharge planning is the most critical. Mr. U had 2 previous psychiatric admissions for alcohol abuse and suicidal ideation. He is a chronic suicide risk who becomes high risk when intoxicated.

Discharge planning begins at admission and is refined during the patient’s stay. Before a patient is discharged, a final post-discharge treatment and aftercare plan is necessary. After discharge, suicide risk increases as the intensity of treatment decreases.8

The patient’s willingness to cooperate with discharge and aftercare planning is critical in establishing contact with follow-up treat­ers. The treatment team should structure the follow-up plan to encourage compliance. For example, psychotic patients at risk of suicide who have a history of stopping medications after discharge can be given a long-acting IM antipsychotic that will last until they reach aftercare. Patients with comorbid drug and alcohol abuse disorders are referred to agen­cies equipped to manage dual-diagnosis patients.

Psychiatrists’ ability to ensure follow-up treatment is limited, a fact that must be acknowledged by the psychiatric and legal communities. Beyond patient stabilization, a clinician’s options to bring about posi­tive changes can be limited or nonexistent. Also, the patient’s failure to adhere to post-discharge plans and treatment often leads to rehospitalization, hopelessness, and greater suicide risk.

Psychiatric patients at moderate or moderate-to-high risk for suicide increas­ingly are treated in outpatient settings. It is the responsibility of the clinician and the treatment team to competently hand off the patient to appropriate outpatient aftercare. With the patient’s permission, the psychia­trist or social worker should call the follow-up agency or therapist before discharge to provide information about the patient’s diagnosis, treatment, and hospital course.

Last, follow-up appointments should be made as close to the time of discharge as possible. Suicide often occurs on the first day after discharge.3

 

 

Bottom Line
Fully commit time and effort to the ongoing assessment, treatment, and management of patients at suicide risk. Suicide risk assessment is a process, not an event. Conduct a suicide risk assessment at important clinical junctures (eg, initial evaluation, discharge, changing observation levels). Contemporaneously, document suicide risk assessments. This self-assessment helps clinicians gauge their strengths and identify skills that need further development.

Disclosure
Dr. Simon reports no financial relationship with any company whose products are mentioned in this article or with manufacturers of competing products.

Dr. Simon is the co-editor of The American Psychiatry Publishing textbook of Suicide Assessment and Management, 2nd edition, from which this article is adapted, by permission of the publisher, American Psychiatry Publishing, Inc. ©2012.

References


1. Simon RI. Gun safety management with patients at risk for suicide. Suicide Life Threat Behav. 2007;37(5):518-526.
2. Qin P, Nordenoft M. Suicide risk in relation to psychiatric hospitalization: evidence based on longitudinal registers. Arch Gen Psychiatry. 2005;62(4):427-432.
3. Appleby L, Shaw J, Amos T, et al. Suicide within 12 months of contact with mental health services: national clinical survey. BMJ. 1999;318(7193):1235-1239.
4. Meehan J, Kapur N, Hunt IM, et al. Suicide in mental health in-patients within 3 months of discharge. National clinical survey. Br J Psychiatry. 2006;188:129-134.
5. Simon RI. Preventing patient suicide: clinical assessment and management. Arlington, VA: American Psychiatric Publishing, Inc.; 2011.
6. Harris CE, Barraclough B. Suicide as an outcome for mental disorders. A meta-analysis. Br J Psychiatry. 1997;170:205-228.
7. Simon RI, Shuman DW. Clinical manual of psychiatry and law. Arlington, VA: American Psychiatric Publishing, Inc.; 2007.
8. Appleby L, Dennehy JA, Thomas CS, et al. Aftercare and clinical characteristics of people with mental illness who commit suicide: a case-control study. Lancet. 1999;353(9162):1397-1400.

References


1. Simon RI. Gun safety management with patients at risk for suicide. Suicide Life Threat Behav. 2007;37(5):518-526.
2. Qin P, Nordenoft M. Suicide risk in relation to psychiatric hospitalization: evidence based on longitudinal registers. Arch Gen Psychiatry. 2005;62(4):427-432.
3. Appleby L, Shaw J, Amos T, et al. Suicide within 12 months of contact with mental health services: national clinical survey. BMJ. 1999;318(7193):1235-1239.
4. Meehan J, Kapur N, Hunt IM, et al. Suicide in mental health in-patients within 3 months of discharge. National clinical survey. Br J Psychiatry. 2006;188:129-134.
5. Simon RI. Preventing patient suicide: clinical assessment and management. Arlington, VA: American Psychiatric Publishing, Inc.; 2011.
6. Harris CE, Barraclough B. Suicide as an outcome for mental disorders. A meta-analysis. Br J Psychiatry. 1997;170:205-228.
7. Simon RI, Shuman DW. Clinical manual of psychiatry and law. Arlington, VA: American Psychiatric Publishing, Inc.; 2007.
8. Appleby L, Dennehy JA, Thomas CS, et al. Aftercare and clinical characteristics of people with mental illness who commit suicide: a case-control study. Lancet. 1999;353(9162):1397-1400.

Issue
Current Psychiatry - 13(11)
Issue
Current Psychiatry - 13(11)
Page Number
21-22, 24-25, 32
Page Number
21-22, 24-25, 32
Publications
Publications
Topics
Article Type
Display Headline
Suicide assessment and management self-test: How do you score?
Display Headline
Suicide assessment and management self-test: How do you score?
Legacy Keywords
suicide risk assessment, suicide, depression, guns, involuntary hospitalization, hospitalization
Legacy Keywords
suicide risk assessment, suicide, depression, guns, involuntary hospitalization, hospitalization
Sections
Article Source

PURLs Copyright

Inside the Article

Article PDF Media

How do you score on this self-assessment of suicide risk management?: First of 2 parts

Article Type
Changed
Thu, 03/28/2019 - 15:39
Display Headline
How do you score on this self-assessment of suicide risk management?: First of 2 parts

The assessment and management of suicide risk are com­plex and difficult tasks that raise clinical issues without clear-cut, easy answers. This case-based, multiple-choice self-assessment with accompanying commentaries is a teach­ing instrument that I designed to enhance a clinician’s ability to provide care for patients at risk for suicide. Part 1 of this article poses 8 of the 15 questions; the balance of questions will appear in Part 2, in the November 2014 issue of Current Psychiatry.

The questions and commentaries in this self-assessment orig­inate in the referenced work of others and my clinical experi­ence. Therefore, I use the preferred “best response” option—not the customary and more restrictive “correct answer” format.

How do you score?


Question 1
Mr. J, age 34, is a professional basketball player complaining of weight loss, early morning waking, and a dysphoric mood last­ing for 1 month. His performance on the basketball court has declined and his wife is seeking a separation. He describes “fleet­ing” suicidal thoughts. He has no history of suicide attempts or depression. The patient does not abuse alcohol or drugs.

The initial assessment approach is to:
   a) obtain a suicide prevention contract
   b) assess suicide risk and protective factors
   c) determine the cause of Mr. J’s depression
   d) have Mr. J complete a suicide risk self-assessment form
   e) contact his wife for additional history


The best response option is B
Suicide prevention contracts do not prevent suicide.1 Contacting the patient’s wife may be an option at a later stage of evaluation or treat­ment, if Mr. J grants permission. Determining the cause of his depression likely will require ongoing work up. Assessing suicide risk fac­tors without also looking at protective factors is a common error. A comprehensive suicide risk assessment evaluation requires evaluat­ing both risk and protective factors.2,3 Suicide risk assessment forms often omit questions about protective factors.4 Do not rely on self-assessment suicide risk forms because they are dependent on the patient’s truthfulness. Patients who are determined to commit sui­cide might regard the psychiatrist and other mental health professionals as the enemy.5


Question 2

Ms. P, a 56-year-old, single schoolteacher, is admitted to a psychiatric unit for severe depres­sion and suicidal ideation without a plan. She is devoutly religious, stating, “I won’t kill myself, because I don’t want to go to hell.” Ms. P attends religious services regularly. She has a history of chronic recurrent depression with suicidal ide­ation and no history of suicide attempts. You suspect a diagnosis of bipolar II disorder.


In assessing religious affiliation as a protective factor against suicide, you should consider:

   a) the nature of the patient’s religious conviction
   b) the religion’s stated position on suicide
   c) severity of the patient’s illness
   d) presence of delusional religious beliefs
   e) all of the above


The best response option is E
Dervic et al6 evaluated 371 depressed inpa­tients according to their religious or non-religious affiliation. Patients with no religious affiliation made significantly more suicide attempts, had more first-degree relatives who committed suicide, were younger, were less likely to be married or have children, and had fewer contacts with family members.

In general, religious affiliation is a protec­tive factor against suicide but may not be a protective factor in an individual patient. Religious affiliation, similar to other pre­summed general protective factors, requires further scrutiny. Avoid making assumptions. For example, a depressed, devoutly religious patient may curse God for abandonment. A patient with bipolar disorder may believe that God would forgive her for committing suicide. A presumed protective factor may not be protective or might even be a risk fac­tor, such as psychotic patients with religious delusions.

Abrahamic religions—ie, Judaism, Christianity, and Islam—prohibit sui­cide. Severe mental illness, however, can overcome the strongest religious prohibi­tions against suicide, including the fear of eternal damnation. For many psychiatric patients, religious affiliations and beliefs are protective factors against suicide, but only relatively. No protective factor against suicide, however strong, provides absolute protection against suicide. Moreover, other risk and protective factors also must be assessed comprehensively.


Question 3
Mr. W, age 18, is admitted to an inpatient psy­chiatric unit with severe agitation, thought disorder, disorganization, and auditory hal­lucinations. He is threatening to jump from a nearby building. He has no history of sub­stance abuse.

The psychiatrist conducts a comprehen­sive suicide risk assessment that includes the patient’s psychiatric diagnosis as a risk factor.

Which psychiatric disorder has the highest associated suicide mortality rate?
   a) schizophrenia
   b) eating disorders
   c) bipolar disorder
   d) major depressive disorder
   e) borderline personality disorder


The best response option is B
Harris and Barraclough (Table)7 calculated the standardized mortality ratio (SMR) for suicide among psychiat­ric disorders. SMR is calculated by divid­ing observed mortality by suicide by the expected mortality by suicide in the general population. Every psychiatric disorder in their study, except for mental retardation, was associated with a varying degree of suicide risk. Eating disorders had the high­est SMR. The patient’s psychiatric diagno­sis is a risk factor that informs the clinician’s suicide risk assessment.



Question 4
Mr. Z, a 64-year-old, recently divorced lawyer, is admitted to the psychiatric unit from the emergency room. His colleagues brought Mr. Z to the emergency room because of his sui­cide threats.

 

 

On the unit, Mr. Z denies suicidal ideation, plan, or intent. Agitation and suspiciousness are prominent. He refuses to authorize staff to contact his colleagues, his ex-wife, and other family members. Mr. Z demands imme­diate discharge and forbids contact with his outpatient psychotherapist. He is placed on 72-hour hold as a conditional voluntary admission.


The clinician should:

   a) contact Mr. Z’s family, as an emergency exception to confidentiality
   b) e-mail his family members with questions
   c) contact the patient’s psychotherapist as permitted by the Health Insurance Portability and Accountability Act of 1996 (HIPAA)
   d) try to develop a therapeutic alliance with Mr. Z
   e) none of the above


The best response option is C

HIPAA permits psychiatrists and other health care providers who are treating the same patient to communicate with each other about medical treatment without obtaining permission from the patient.8 However, mental health professionals can­not share psychotherapy notes without a patient’s consent, except when legally required, such as reporting abuse or duty to warn. This is the most expeditious and productive way of obtaining essential clin­ical information. E-mail merely changes the mode of unauthorized communication with significant others.

Mr. Z is agitated and suspicious, and developing a therapeutic alliance would require time. It is necessary to gather infor­mation about his psychiatric condition as soon as possible. An emergency exception to maintaining confidentiality is another option.9 The definition of emergency var­ies among jurisdictions. Consulting with a knowledgeable attorney may be neces­sary, but it usually takes time. Ethically, it is permissible to breach confidentiality to protect the suicidal patient.10


Question 5

Mr. G, a 42-year-old engineer, is re-hospitalized after a failed hanging attempt. Initially, he is profoundly depressed but improves sud­denly and requests discharge. The psychiatrist and clinical staff are perplexed. Is the sudden improvement real or feigned?

The treatment team should consider all of the following options except:
   
a) obtain records of earlier hospitalizations
   b) check collateral sources of information
   c) assess Mr. G’s compliance with treatment
   d) obtain psychological testing to evaluate Mr. G’s honesty
   e) determine whether behavioral signs of depression are present


The best response option is D
Short length of hospital stay makes it diffi­cult to assess sudden patient improvement.11 Real improvement in a high-risk suicidal patient is a process, even when it occurs quickly. Feigned improvement is an event. Obtaining patient information from collat­eral sources is crucial. Sudden improvement might be caused by the patient’s resolve to complete suicide. Identifying behavioral risk factors associated with psychiatric disorders informs the clinician’s systematic suicide risk assessment of a guarded or dissimula­tive patient. Psychological testing will take critical time and is not a substitute for careful clinical assessment.


Question 6

In mid-winter, Ms. M, a 42-year-old homeless woman, is seen in the emergency room of a general hospital. She complains of depres­sion and auditory hallucinations command­ing her to commit suicide. Ms. M has 5 earlier admissions to the psychiatry unit for similar complaints.

The psychiatrist conducts a compre­hensive suicide risk assessment. Acute and chronic risk factors for suicide are identified. Protective factors also are assessed. The psy­chiatrist weighs and synthesizes risk and pro­tective factors into an overall assessment of Ms. M’s suicide risk.

The main purpose of suicide risk assessment is to:  
   a) predict the likelihood of suicide  
   b) determine imminence of suicide  
   c) inform patient treatment and safety management  
   d) identify malingered suicidal ideation  
   e) provide a legal defense against a mal­practice claim


The best response option is C
Suicide cannot be predicted.12 The term imminent suicide is a veiled attempt to pre­dict when a patient will attempt suicide.13 The process of a comprehensive or system­atic suicide risk assessment encompasses identification, analysis, and synthesis of risk and protective factors that inform the treatment and safety management of the patient.3 The overall suicide assessment is a clinical judgment call that determines risk along a continuum of low to high. In Ms. M’s case, comprehensive suicide risk assessment will assist the clinician in deter­mining the patient’s overall suicide risk and make an appropriate disposition. Without a systematic suicide risk assessment meth­odology, the clinician is at the mercy of the pejoratively labeled “frequent flyer” who is looking for sustenance and lodging. The frustrated clinician is left with little choice but to admit the patient.

Although not the main purpose, system­atic suicide risk assessment can help provide a sound legal defense if a suicide malpractice claim is filed against the clinician alleging negligent assessment.14


Question 7
A psychiatrist is treating Mr. S, a 36-year-old computer analyst, with once-a-week psycho­therapy and medication management for panic and depressive symptoms that emerged abruptly after the break-up of a romantic rela­tionship. Mr. S is using alcohol to sleep. He reports occasional suicidal ideation but no plan. He finds the idea of suicide to be morally repugnant. A therapeutic alliance develops.

 

 

The psychiatrist is concerned about Mr. S’s suicide risk and the need for hospi­talization. The psychiatrist performs a sys­tematic suicide risk assessment that includes identification of individual and evidence-based protective factors. For example, Mr. S continued to pursue his interests and to par­ticipate in civil causes. The overall suicide risk  is determined by the assessment of individ­ual and evidence-based protective factors.


All of the following options are evidence-based protective factors except:

   a) therapeutic alliance
   b) survival and coping beliefs
   c) responsibility to family
   d) fear of suicide
   e) moral objections to suicide


The best response option is A

Clinical consensus holds that the thera­peutic alliance is an important protec­tive factor against suicide. However, no evidence-based research supports or refutes this widely held belief among clinicians.

Linehan et al15 developed the Reasons for Living Inventory, a self-report instru­ment that identifies 6 subscales:
   • survival and coping beliefs
   • responsibility to family
   • child-related concerns
   • fear of suicide
   • fear of social disapproval
   • moral objections to suicide.

Survival and coping beliefs, responsi­bility to family, and child-related concerns were useful in differentiating between suicidal and non-suicidal individuals. Malone et al16 administered the Reasons for Living Inventory to 84 inpatients with major depression; 45 had attempted sui­cide. Depressed patients who had not attempted suicide demonstrated more sense of responsibility toward family, more fear of social disapproval, more moral objections to suicide, greater survival and coping skills, and greater fear of suicide than patients who attempted suicide. The authors recommended adding the Reasons for Living Inventory to the assessment of patients at risk for suicide.


Question 8
A 38-year-old mother of a newborn child is admitted to the psychiatric unit after expressing suicidal thoughts to her husband. She has been hospitalized previously after a hypomanic episode and severe depres­sion; she has no history of suicide attempts. A psychiatrist diagnoses bipolar II disorder (recurrent major episodes with hypomanic episodes). The patient’s maternal aunt has bipolar disorder. Her paternal grandfather committed suicide.

The psychiatrist conducts a systematic suicide risk assessment and determines the patient is at high risk of suicide. He considers a suicide-risk reduction drug.


Which one of the following drugs has been shown to reduce suicide and suicide attempts in bipolar II patients?

   a) clozapine
   b) clonazepam
   c) lorazepam
   d) lithium
   e) quetiapine


The best response option is D
Prospective, randomized and controlled trials consistently have found lower rates of completed suicides and suicide attempts during lithium maintenance treatments for patients with bipolar disorder and other major affective disorders.17

Bottom Line
Suicide risk assessment and management are challenging for even experienced clinicians. Suicide risk assessment guides appropriate treatment and management for patients at risk for suicide. This self-assessment helps mental health professionals identify potential gaps in their knowledge and reinforce best practices.

Related Resources
• Simon RI. Passive suicidal ideation: Still a high-risk clinical scenario. Current Psychiatry. 2014;13(3):13-15.
• Simon RI. Suicide rehearsals: A high-risk psychiatric emer­gency. Current Psychiatry. 2012;11(7):28-32.
• Bongar B, Sullivan GR. The suicidal patient: Clinical and legal standards of care. Washington, DC: American Psychological Association; 2013.

Drug Brand Names
Clonazepam • Klonopin              Lorazepam • Ativan
Clozapine • Clozaril                    Quetiapine • Seroquel
Lithium • Eskalith, Lithobid


Disclosure
Dr. Simon reports no financial relationship with any company whose products are mentioned in this article or with manufacturers of competing products.

Adapted with permission from: Simon RI. Preventing patient suicide: clinical assessment and management, Arlington VA: American Psychiatric Publishing; 2011.

Editor’s note: Part 2 of this self-assessment on suicide assessment and management in the November 2014 issue of Current Psychiatry poses 7 additional questions.

References


1. Stanford EJ, Goetz RR, Bloom JD. The No Harm Contract in the emergency assessment of suicide risk. J Clin Psychiatry. 1994;55(8):344-348.
2. Simon RI, Hales RE, eds. Textbook of suicide assessment and management. 2nd ed. Arlington, VA: American Psychiatric Publishing, Inc.; 2012.
3. Practice guidelines for the assessment and treatment of patients with suicidal behaviors [Erratum in Am J Psychiatry. 2004;161(4):776]. Am J Psychiatry. 2003;160(suppl 11):1-60.
4. Simon RI. Suicide risk assessment forms: form over substance? J Am Acad Psychiatry Law. 2009;37(3): 290-293.
5. Resnick PJ. Recognizing that the suicidal patient views you as an ‘adversary.’ Current Psychiatry. 2002;1(1):8.
6. Dervic K, Oquendo MA, Grunebaum MF, et al. Religious affiliation and suicide attempt. Am J Psychiatry. 2004; 161(12):2303-2308.
7. Harris CE, Barraclough B. Suicide as an outcome for mental disorders. A meta-analysis. Br J Psychiatry. 1997;170:205-228.
8. Health insurance portability and accountability act of 1996. Pub L No. 104-191.
9. Simon RI, Shuman DW. Clinical manual of psychiatry and law. Arlington, VA: American Psychiatric Publishing, Inc; 2007.
10. American Psychiatric Association. Principles of medical ethics with annotations especially applicable to psychiatry. Section 4, annotation 8. Washington, DC: American Psychiatric Publishing, Inc; 2001.
11. Simon RI, Gutheil TG. Sudden improvement in high-risk suicidal patients: should it be trusted? Psych Serv. 2009; 60(3):387-389.
12. Pokorny AD. Prediction of suicide in psychiatric patients. Report of a prospective study. Arch Gen Psychiatry. 1983; 4(3):249-257.
13. Simon RI. Imminent suicide: the illusion of short-term prediction. Suicide Life Threat Behav. 2006;36(3): 296-301.
14. Simon RI, Shuman DW. Therapeutic risk management of clinical-legal dilemmas: should it be a core competency? J Am Acad Psychiatry Law. 2009;37(2):155-161.
15. Linehan MM, Goodstein JL, Nielsen SL, et al. Reasons for staying alive when you are thinking of killing yourself: the reasons for living inventory. J Consult Clin Psychol. 1983;51(2):276-286.
16. Malone KM, Oquendo MA, Hass GL, et al. Protective factors against suicidal acts in major depression: reasons for living. Am J Psychiatry. 2000;157(7):1084-1088.
17. Baldessarini RJ, Pompili M, Tondo L. Bipolar disorder. In: Simon RI, Hales RE, eds. Textbook of suicide assessment and management. Arlington, VA: American Psychiatric Publishing, Inc; 2006:159-176.

Article PDF
Author and Disclosure Information

Robert I. Simon, MD
Clinical Professor of Psychiatry
Georgetown University School of Medicine
Washington, DC

Issue
Current Psychiatry - 13(10)
Publications
Topics
Page Number
26-28, 30-32
Legacy Keywords
suicide, suicide risk, suicide assessment, suicide risk assessment
Sections
Author and Disclosure Information

Robert I. Simon, MD
Clinical Professor of Psychiatry
Georgetown University School of Medicine
Washington, DC

Author and Disclosure Information

Robert I. Simon, MD
Clinical Professor of Psychiatry
Georgetown University School of Medicine
Washington, DC

Article PDF
Article PDF
Related Articles

The assessment and management of suicide risk are com­plex and difficult tasks that raise clinical issues without clear-cut, easy answers. This case-based, multiple-choice self-assessment with accompanying commentaries is a teach­ing instrument that I designed to enhance a clinician’s ability to provide care for patients at risk for suicide. Part 1 of this article poses 8 of the 15 questions; the balance of questions will appear in Part 2, in the November 2014 issue of Current Psychiatry.

The questions and commentaries in this self-assessment orig­inate in the referenced work of others and my clinical experi­ence. Therefore, I use the preferred “best response” option—not the customary and more restrictive “correct answer” format.

How do you score?


Question 1
Mr. J, age 34, is a professional basketball player complaining of weight loss, early morning waking, and a dysphoric mood last­ing for 1 month. His performance on the basketball court has declined and his wife is seeking a separation. He describes “fleet­ing” suicidal thoughts. He has no history of suicide attempts or depression. The patient does not abuse alcohol or drugs.

The initial assessment approach is to:
   a) obtain a suicide prevention contract
   b) assess suicide risk and protective factors
   c) determine the cause of Mr. J’s depression
   d) have Mr. J complete a suicide risk self-assessment form
   e) contact his wife for additional history


The best response option is B
Suicide prevention contracts do not prevent suicide.1 Contacting the patient’s wife may be an option at a later stage of evaluation or treat­ment, if Mr. J grants permission. Determining the cause of his depression likely will require ongoing work up. Assessing suicide risk fac­tors without also looking at protective factors is a common error. A comprehensive suicide risk assessment evaluation requires evaluat­ing both risk and protective factors.2,3 Suicide risk assessment forms often omit questions about protective factors.4 Do not rely on self-assessment suicide risk forms because they are dependent on the patient’s truthfulness. Patients who are determined to commit sui­cide might regard the psychiatrist and other mental health professionals as the enemy.5


Question 2

Ms. P, a 56-year-old, single schoolteacher, is admitted to a psychiatric unit for severe depres­sion and suicidal ideation without a plan. She is devoutly religious, stating, “I won’t kill myself, because I don’t want to go to hell.” Ms. P attends religious services regularly. She has a history of chronic recurrent depression with suicidal ide­ation and no history of suicide attempts. You suspect a diagnosis of bipolar II disorder.


In assessing religious affiliation as a protective factor against suicide, you should consider:

   a) the nature of the patient’s religious conviction
   b) the religion’s stated position on suicide
   c) severity of the patient’s illness
   d) presence of delusional religious beliefs
   e) all of the above


The best response option is E
Dervic et al6 evaluated 371 depressed inpa­tients according to their religious or non-religious affiliation. Patients with no religious affiliation made significantly more suicide attempts, had more first-degree relatives who committed suicide, were younger, were less likely to be married or have children, and had fewer contacts with family members.

In general, religious affiliation is a protec­tive factor against suicide but may not be a protective factor in an individual patient. Religious affiliation, similar to other pre­summed general protective factors, requires further scrutiny. Avoid making assumptions. For example, a depressed, devoutly religious patient may curse God for abandonment. A patient with bipolar disorder may believe that God would forgive her for committing suicide. A presumed protective factor may not be protective or might even be a risk fac­tor, such as psychotic patients with religious delusions.

Abrahamic religions—ie, Judaism, Christianity, and Islam—prohibit sui­cide. Severe mental illness, however, can overcome the strongest religious prohibi­tions against suicide, including the fear of eternal damnation. For many psychiatric patients, religious affiliations and beliefs are protective factors against suicide, but only relatively. No protective factor against suicide, however strong, provides absolute protection against suicide. Moreover, other risk and protective factors also must be assessed comprehensively.


Question 3
Mr. W, age 18, is admitted to an inpatient psy­chiatric unit with severe agitation, thought disorder, disorganization, and auditory hal­lucinations. He is threatening to jump from a nearby building. He has no history of sub­stance abuse.

The psychiatrist conducts a comprehen­sive suicide risk assessment that includes the patient’s psychiatric diagnosis as a risk factor.

Which psychiatric disorder has the highest associated suicide mortality rate?
   a) schizophrenia
   b) eating disorders
   c) bipolar disorder
   d) major depressive disorder
   e) borderline personality disorder


The best response option is B
Harris and Barraclough (Table)7 calculated the standardized mortality ratio (SMR) for suicide among psychiat­ric disorders. SMR is calculated by divid­ing observed mortality by suicide by the expected mortality by suicide in the general population. Every psychiatric disorder in their study, except for mental retardation, was associated with a varying degree of suicide risk. Eating disorders had the high­est SMR. The patient’s psychiatric diagno­sis is a risk factor that informs the clinician’s suicide risk assessment.



Question 4
Mr. Z, a 64-year-old, recently divorced lawyer, is admitted to the psychiatric unit from the emergency room. His colleagues brought Mr. Z to the emergency room because of his sui­cide threats.

 

 

On the unit, Mr. Z denies suicidal ideation, plan, or intent. Agitation and suspiciousness are prominent. He refuses to authorize staff to contact his colleagues, his ex-wife, and other family members. Mr. Z demands imme­diate discharge and forbids contact with his outpatient psychotherapist. He is placed on 72-hour hold as a conditional voluntary admission.


The clinician should:

   a) contact Mr. Z’s family, as an emergency exception to confidentiality
   b) e-mail his family members with questions
   c) contact the patient’s psychotherapist as permitted by the Health Insurance Portability and Accountability Act of 1996 (HIPAA)
   d) try to develop a therapeutic alliance with Mr. Z
   e) none of the above


The best response option is C

HIPAA permits psychiatrists and other health care providers who are treating the same patient to communicate with each other about medical treatment without obtaining permission from the patient.8 However, mental health professionals can­not share psychotherapy notes without a patient’s consent, except when legally required, such as reporting abuse or duty to warn. This is the most expeditious and productive way of obtaining essential clin­ical information. E-mail merely changes the mode of unauthorized communication with significant others.

Mr. Z is agitated and suspicious, and developing a therapeutic alliance would require time. It is necessary to gather infor­mation about his psychiatric condition as soon as possible. An emergency exception to maintaining confidentiality is another option.9 The definition of emergency var­ies among jurisdictions. Consulting with a knowledgeable attorney may be neces­sary, but it usually takes time. Ethically, it is permissible to breach confidentiality to protect the suicidal patient.10


Question 5

Mr. G, a 42-year-old engineer, is re-hospitalized after a failed hanging attempt. Initially, he is profoundly depressed but improves sud­denly and requests discharge. The psychiatrist and clinical staff are perplexed. Is the sudden improvement real or feigned?

The treatment team should consider all of the following options except:
   
a) obtain records of earlier hospitalizations
   b) check collateral sources of information
   c) assess Mr. G’s compliance with treatment
   d) obtain psychological testing to evaluate Mr. G’s honesty
   e) determine whether behavioral signs of depression are present


The best response option is D
Short length of hospital stay makes it diffi­cult to assess sudden patient improvement.11 Real improvement in a high-risk suicidal patient is a process, even when it occurs quickly. Feigned improvement is an event. Obtaining patient information from collat­eral sources is crucial. Sudden improvement might be caused by the patient’s resolve to complete suicide. Identifying behavioral risk factors associated with psychiatric disorders informs the clinician’s systematic suicide risk assessment of a guarded or dissimula­tive patient. Psychological testing will take critical time and is not a substitute for careful clinical assessment.


Question 6

In mid-winter, Ms. M, a 42-year-old homeless woman, is seen in the emergency room of a general hospital. She complains of depres­sion and auditory hallucinations command­ing her to commit suicide. Ms. M has 5 earlier admissions to the psychiatry unit for similar complaints.

The psychiatrist conducts a compre­hensive suicide risk assessment. Acute and chronic risk factors for suicide are identified. Protective factors also are assessed. The psy­chiatrist weighs and synthesizes risk and pro­tective factors into an overall assessment of Ms. M’s suicide risk.

The main purpose of suicide risk assessment is to:  
   a) predict the likelihood of suicide  
   b) determine imminence of suicide  
   c) inform patient treatment and safety management  
   d) identify malingered suicidal ideation  
   e) provide a legal defense against a mal­practice claim


The best response option is C
Suicide cannot be predicted.12 The term imminent suicide is a veiled attempt to pre­dict when a patient will attempt suicide.13 The process of a comprehensive or system­atic suicide risk assessment encompasses identification, analysis, and synthesis of risk and protective factors that inform the treatment and safety management of the patient.3 The overall suicide assessment is a clinical judgment call that determines risk along a continuum of low to high. In Ms. M’s case, comprehensive suicide risk assessment will assist the clinician in deter­mining the patient’s overall suicide risk and make an appropriate disposition. Without a systematic suicide risk assessment meth­odology, the clinician is at the mercy of the pejoratively labeled “frequent flyer” who is looking for sustenance and lodging. The frustrated clinician is left with little choice but to admit the patient.

Although not the main purpose, system­atic suicide risk assessment can help provide a sound legal defense if a suicide malpractice claim is filed against the clinician alleging negligent assessment.14


Question 7
A psychiatrist is treating Mr. S, a 36-year-old computer analyst, with once-a-week psycho­therapy and medication management for panic and depressive symptoms that emerged abruptly after the break-up of a romantic rela­tionship. Mr. S is using alcohol to sleep. He reports occasional suicidal ideation but no plan. He finds the idea of suicide to be morally repugnant. A therapeutic alliance develops.

 

 

The psychiatrist is concerned about Mr. S’s suicide risk and the need for hospi­talization. The psychiatrist performs a sys­tematic suicide risk assessment that includes identification of individual and evidence-based protective factors. For example, Mr. S continued to pursue his interests and to par­ticipate in civil causes. The overall suicide risk  is determined by the assessment of individ­ual and evidence-based protective factors.


All of the following options are evidence-based protective factors except:

   a) therapeutic alliance
   b) survival and coping beliefs
   c) responsibility to family
   d) fear of suicide
   e) moral objections to suicide


The best response option is A

Clinical consensus holds that the thera­peutic alliance is an important protec­tive factor against suicide. However, no evidence-based research supports or refutes this widely held belief among clinicians.

Linehan et al15 developed the Reasons for Living Inventory, a self-report instru­ment that identifies 6 subscales:
   • survival and coping beliefs
   • responsibility to family
   • child-related concerns
   • fear of suicide
   • fear of social disapproval
   • moral objections to suicide.

Survival and coping beliefs, responsi­bility to family, and child-related concerns were useful in differentiating between suicidal and non-suicidal individuals. Malone et al16 administered the Reasons for Living Inventory to 84 inpatients with major depression; 45 had attempted sui­cide. Depressed patients who had not attempted suicide demonstrated more sense of responsibility toward family, more fear of social disapproval, more moral objections to suicide, greater survival and coping skills, and greater fear of suicide than patients who attempted suicide. The authors recommended adding the Reasons for Living Inventory to the assessment of patients at risk for suicide.


Question 8
A 38-year-old mother of a newborn child is admitted to the psychiatric unit after expressing suicidal thoughts to her husband. She has been hospitalized previously after a hypomanic episode and severe depres­sion; she has no history of suicide attempts. A psychiatrist diagnoses bipolar II disorder (recurrent major episodes with hypomanic episodes). The patient’s maternal aunt has bipolar disorder. Her paternal grandfather committed suicide.

The psychiatrist conducts a systematic suicide risk assessment and determines the patient is at high risk of suicide. He considers a suicide-risk reduction drug.


Which one of the following drugs has been shown to reduce suicide and suicide attempts in bipolar II patients?

   a) clozapine
   b) clonazepam
   c) lorazepam
   d) lithium
   e) quetiapine


The best response option is D
Prospective, randomized and controlled trials consistently have found lower rates of completed suicides and suicide attempts during lithium maintenance treatments for patients with bipolar disorder and other major affective disorders.17

Bottom Line
Suicide risk assessment and management are challenging for even experienced clinicians. Suicide risk assessment guides appropriate treatment and management for patients at risk for suicide. This self-assessment helps mental health professionals identify potential gaps in their knowledge and reinforce best practices.

Related Resources
• Simon RI. Passive suicidal ideation: Still a high-risk clinical scenario. Current Psychiatry. 2014;13(3):13-15.
• Simon RI. Suicide rehearsals: A high-risk psychiatric emer­gency. Current Psychiatry. 2012;11(7):28-32.
• Bongar B, Sullivan GR. The suicidal patient: Clinical and legal standards of care. Washington, DC: American Psychological Association; 2013.

Drug Brand Names
Clonazepam • Klonopin              Lorazepam • Ativan
Clozapine • Clozaril                    Quetiapine • Seroquel
Lithium • Eskalith, Lithobid


Disclosure
Dr. Simon reports no financial relationship with any company whose products are mentioned in this article or with manufacturers of competing products.

Adapted with permission from: Simon RI. Preventing patient suicide: clinical assessment and management, Arlington VA: American Psychiatric Publishing; 2011.

Editor’s note: Part 2 of this self-assessment on suicide assessment and management in the November 2014 issue of Current Psychiatry poses 7 additional questions.

The assessment and management of suicide risk are com­plex and difficult tasks that raise clinical issues without clear-cut, easy answers. This case-based, multiple-choice self-assessment with accompanying commentaries is a teach­ing instrument that I designed to enhance a clinician’s ability to provide care for patients at risk for suicide. Part 1 of this article poses 8 of the 15 questions; the balance of questions will appear in Part 2, in the November 2014 issue of Current Psychiatry.

The questions and commentaries in this self-assessment orig­inate in the referenced work of others and my clinical experi­ence. Therefore, I use the preferred “best response” option—not the customary and more restrictive “correct answer” format.

How do you score?


Question 1
Mr. J, age 34, is a professional basketball player complaining of weight loss, early morning waking, and a dysphoric mood last­ing for 1 month. His performance on the basketball court has declined and his wife is seeking a separation. He describes “fleet­ing” suicidal thoughts. He has no history of suicide attempts or depression. The patient does not abuse alcohol or drugs.

The initial assessment approach is to:
   a) obtain a suicide prevention contract
   b) assess suicide risk and protective factors
   c) determine the cause of Mr. J’s depression
   d) have Mr. J complete a suicide risk self-assessment form
   e) contact his wife for additional history


The best response option is B
Suicide prevention contracts do not prevent suicide.1 Contacting the patient’s wife may be an option at a later stage of evaluation or treat­ment, if Mr. J grants permission. Determining the cause of his depression likely will require ongoing work up. Assessing suicide risk fac­tors without also looking at protective factors is a common error. A comprehensive suicide risk assessment evaluation requires evaluat­ing both risk and protective factors.2,3 Suicide risk assessment forms often omit questions about protective factors.4 Do not rely on self-assessment suicide risk forms because they are dependent on the patient’s truthfulness. Patients who are determined to commit sui­cide might regard the psychiatrist and other mental health professionals as the enemy.5


Question 2

Ms. P, a 56-year-old, single schoolteacher, is admitted to a psychiatric unit for severe depres­sion and suicidal ideation without a plan. She is devoutly religious, stating, “I won’t kill myself, because I don’t want to go to hell.” Ms. P attends religious services regularly. She has a history of chronic recurrent depression with suicidal ide­ation and no history of suicide attempts. You suspect a diagnosis of bipolar II disorder.


In assessing religious affiliation as a protective factor against suicide, you should consider:

   a) the nature of the patient’s religious conviction
   b) the religion’s stated position on suicide
   c) severity of the patient’s illness
   d) presence of delusional religious beliefs
   e) all of the above


The best response option is E
Dervic et al6 evaluated 371 depressed inpa­tients according to their religious or non-religious affiliation. Patients with no religious affiliation made significantly more suicide attempts, had more first-degree relatives who committed suicide, were younger, were less likely to be married or have children, and had fewer contacts with family members.

In general, religious affiliation is a protec­tive factor against suicide but may not be a protective factor in an individual patient. Religious affiliation, similar to other pre­summed general protective factors, requires further scrutiny. Avoid making assumptions. For example, a depressed, devoutly religious patient may curse God for abandonment. A patient with bipolar disorder may believe that God would forgive her for committing suicide. A presumed protective factor may not be protective or might even be a risk fac­tor, such as psychotic patients with religious delusions.

Abrahamic religions—ie, Judaism, Christianity, and Islam—prohibit sui­cide. Severe mental illness, however, can overcome the strongest religious prohibi­tions against suicide, including the fear of eternal damnation. For many psychiatric patients, religious affiliations and beliefs are protective factors against suicide, but only relatively. No protective factor against suicide, however strong, provides absolute protection against suicide. Moreover, other risk and protective factors also must be assessed comprehensively.


Question 3
Mr. W, age 18, is admitted to an inpatient psy­chiatric unit with severe agitation, thought disorder, disorganization, and auditory hal­lucinations. He is threatening to jump from a nearby building. He has no history of sub­stance abuse.

The psychiatrist conducts a comprehen­sive suicide risk assessment that includes the patient’s psychiatric diagnosis as a risk factor.

Which psychiatric disorder has the highest associated suicide mortality rate?
   a) schizophrenia
   b) eating disorders
   c) bipolar disorder
   d) major depressive disorder
   e) borderline personality disorder


The best response option is B
Harris and Barraclough (Table)7 calculated the standardized mortality ratio (SMR) for suicide among psychiat­ric disorders. SMR is calculated by divid­ing observed mortality by suicide by the expected mortality by suicide in the general population. Every psychiatric disorder in their study, except for mental retardation, was associated with a varying degree of suicide risk. Eating disorders had the high­est SMR. The patient’s psychiatric diagno­sis is a risk factor that informs the clinician’s suicide risk assessment.



Question 4
Mr. Z, a 64-year-old, recently divorced lawyer, is admitted to the psychiatric unit from the emergency room. His colleagues brought Mr. Z to the emergency room because of his sui­cide threats.

 

 

On the unit, Mr. Z denies suicidal ideation, plan, or intent. Agitation and suspiciousness are prominent. He refuses to authorize staff to contact his colleagues, his ex-wife, and other family members. Mr. Z demands imme­diate discharge and forbids contact with his outpatient psychotherapist. He is placed on 72-hour hold as a conditional voluntary admission.


The clinician should:

   a) contact Mr. Z’s family, as an emergency exception to confidentiality
   b) e-mail his family members with questions
   c) contact the patient’s psychotherapist as permitted by the Health Insurance Portability and Accountability Act of 1996 (HIPAA)
   d) try to develop a therapeutic alliance with Mr. Z
   e) none of the above


The best response option is C

HIPAA permits psychiatrists and other health care providers who are treating the same patient to communicate with each other about medical treatment without obtaining permission from the patient.8 However, mental health professionals can­not share psychotherapy notes without a patient’s consent, except when legally required, such as reporting abuse or duty to warn. This is the most expeditious and productive way of obtaining essential clin­ical information. E-mail merely changes the mode of unauthorized communication with significant others.

Mr. Z is agitated and suspicious, and developing a therapeutic alliance would require time. It is necessary to gather infor­mation about his psychiatric condition as soon as possible. An emergency exception to maintaining confidentiality is another option.9 The definition of emergency var­ies among jurisdictions. Consulting with a knowledgeable attorney may be neces­sary, but it usually takes time. Ethically, it is permissible to breach confidentiality to protect the suicidal patient.10


Question 5

Mr. G, a 42-year-old engineer, is re-hospitalized after a failed hanging attempt. Initially, he is profoundly depressed but improves sud­denly and requests discharge. The psychiatrist and clinical staff are perplexed. Is the sudden improvement real or feigned?

The treatment team should consider all of the following options except:
   
a) obtain records of earlier hospitalizations
   b) check collateral sources of information
   c) assess Mr. G’s compliance with treatment
   d) obtain psychological testing to evaluate Mr. G’s honesty
   e) determine whether behavioral signs of depression are present


The best response option is D
Short length of hospital stay makes it diffi­cult to assess sudden patient improvement.11 Real improvement in a high-risk suicidal patient is a process, even when it occurs quickly. Feigned improvement is an event. Obtaining patient information from collat­eral sources is crucial. Sudden improvement might be caused by the patient’s resolve to complete suicide. Identifying behavioral risk factors associated with psychiatric disorders informs the clinician’s systematic suicide risk assessment of a guarded or dissimula­tive patient. Psychological testing will take critical time and is not a substitute for careful clinical assessment.


Question 6

In mid-winter, Ms. M, a 42-year-old homeless woman, is seen in the emergency room of a general hospital. She complains of depres­sion and auditory hallucinations command­ing her to commit suicide. Ms. M has 5 earlier admissions to the psychiatry unit for similar complaints.

The psychiatrist conducts a compre­hensive suicide risk assessment. Acute and chronic risk factors for suicide are identified. Protective factors also are assessed. The psy­chiatrist weighs and synthesizes risk and pro­tective factors into an overall assessment of Ms. M’s suicide risk.

The main purpose of suicide risk assessment is to:  
   a) predict the likelihood of suicide  
   b) determine imminence of suicide  
   c) inform patient treatment and safety management  
   d) identify malingered suicidal ideation  
   e) provide a legal defense against a mal­practice claim


The best response option is C
Suicide cannot be predicted.12 The term imminent suicide is a veiled attempt to pre­dict when a patient will attempt suicide.13 The process of a comprehensive or system­atic suicide risk assessment encompasses identification, analysis, and synthesis of risk and protective factors that inform the treatment and safety management of the patient.3 The overall suicide assessment is a clinical judgment call that determines risk along a continuum of low to high. In Ms. M’s case, comprehensive suicide risk assessment will assist the clinician in deter­mining the patient’s overall suicide risk and make an appropriate disposition. Without a systematic suicide risk assessment meth­odology, the clinician is at the mercy of the pejoratively labeled “frequent flyer” who is looking for sustenance and lodging. The frustrated clinician is left with little choice but to admit the patient.

Although not the main purpose, system­atic suicide risk assessment can help provide a sound legal defense if a suicide malpractice claim is filed against the clinician alleging negligent assessment.14


Question 7
A psychiatrist is treating Mr. S, a 36-year-old computer analyst, with once-a-week psycho­therapy and medication management for panic and depressive symptoms that emerged abruptly after the break-up of a romantic rela­tionship. Mr. S is using alcohol to sleep. He reports occasional suicidal ideation but no plan. He finds the idea of suicide to be morally repugnant. A therapeutic alliance develops.

 

 

The psychiatrist is concerned about Mr. S’s suicide risk and the need for hospi­talization. The psychiatrist performs a sys­tematic suicide risk assessment that includes identification of individual and evidence-based protective factors. For example, Mr. S continued to pursue his interests and to par­ticipate in civil causes. The overall suicide risk  is determined by the assessment of individ­ual and evidence-based protective factors.


All of the following options are evidence-based protective factors except:

   a) therapeutic alliance
   b) survival and coping beliefs
   c) responsibility to family
   d) fear of suicide
   e) moral objections to suicide


The best response option is A

Clinical consensus holds that the thera­peutic alliance is an important protec­tive factor against suicide. However, no evidence-based research supports or refutes this widely held belief among clinicians.

Linehan et al15 developed the Reasons for Living Inventory, a self-report instru­ment that identifies 6 subscales:
   • survival and coping beliefs
   • responsibility to family
   • child-related concerns
   • fear of suicide
   • fear of social disapproval
   • moral objections to suicide.

Survival and coping beliefs, responsi­bility to family, and child-related concerns were useful in differentiating between suicidal and non-suicidal individuals. Malone et al16 administered the Reasons for Living Inventory to 84 inpatients with major depression; 45 had attempted sui­cide. Depressed patients who had not attempted suicide demonstrated more sense of responsibility toward family, more fear of social disapproval, more moral objections to suicide, greater survival and coping skills, and greater fear of suicide than patients who attempted suicide. The authors recommended adding the Reasons for Living Inventory to the assessment of patients at risk for suicide.


Question 8
A 38-year-old mother of a newborn child is admitted to the psychiatric unit after expressing suicidal thoughts to her husband. She has been hospitalized previously after a hypomanic episode and severe depres­sion; she has no history of suicide attempts. A psychiatrist diagnoses bipolar II disorder (recurrent major episodes with hypomanic episodes). The patient’s maternal aunt has bipolar disorder. Her paternal grandfather committed suicide.

The psychiatrist conducts a systematic suicide risk assessment and determines the patient is at high risk of suicide. He considers a suicide-risk reduction drug.


Which one of the following drugs has been shown to reduce suicide and suicide attempts in bipolar II patients?

   a) clozapine
   b) clonazepam
   c) lorazepam
   d) lithium
   e) quetiapine


The best response option is D
Prospective, randomized and controlled trials consistently have found lower rates of completed suicides and suicide attempts during lithium maintenance treatments for patients with bipolar disorder and other major affective disorders.17

Bottom Line
Suicide risk assessment and management are challenging for even experienced clinicians. Suicide risk assessment guides appropriate treatment and management for patients at risk for suicide. This self-assessment helps mental health professionals identify potential gaps in their knowledge and reinforce best practices.

Related Resources
• Simon RI. Passive suicidal ideation: Still a high-risk clinical scenario. Current Psychiatry. 2014;13(3):13-15.
• Simon RI. Suicide rehearsals: A high-risk psychiatric emer­gency. Current Psychiatry. 2012;11(7):28-32.
• Bongar B, Sullivan GR. The suicidal patient: Clinical and legal standards of care. Washington, DC: American Psychological Association; 2013.

Drug Brand Names
Clonazepam • Klonopin              Lorazepam • Ativan
Clozapine • Clozaril                    Quetiapine • Seroquel
Lithium • Eskalith, Lithobid


Disclosure
Dr. Simon reports no financial relationship with any company whose products are mentioned in this article or with manufacturers of competing products.

Adapted with permission from: Simon RI. Preventing patient suicide: clinical assessment and management, Arlington VA: American Psychiatric Publishing; 2011.

Editor’s note: Part 2 of this self-assessment on suicide assessment and management in the November 2014 issue of Current Psychiatry poses 7 additional questions.

References


1. Stanford EJ, Goetz RR, Bloom JD. The No Harm Contract in the emergency assessment of suicide risk. J Clin Psychiatry. 1994;55(8):344-348.
2. Simon RI, Hales RE, eds. Textbook of suicide assessment and management. 2nd ed. Arlington, VA: American Psychiatric Publishing, Inc.; 2012.
3. Practice guidelines for the assessment and treatment of patients with suicidal behaviors [Erratum in Am J Psychiatry. 2004;161(4):776]. Am J Psychiatry. 2003;160(suppl 11):1-60.
4. Simon RI. Suicide risk assessment forms: form over substance? J Am Acad Psychiatry Law. 2009;37(3): 290-293.
5. Resnick PJ. Recognizing that the suicidal patient views you as an ‘adversary.’ Current Psychiatry. 2002;1(1):8.
6. Dervic K, Oquendo MA, Grunebaum MF, et al. Religious affiliation and suicide attempt. Am J Psychiatry. 2004; 161(12):2303-2308.
7. Harris CE, Barraclough B. Suicide as an outcome for mental disorders. A meta-analysis. Br J Psychiatry. 1997;170:205-228.
8. Health insurance portability and accountability act of 1996. Pub L No. 104-191.
9. Simon RI, Shuman DW. Clinical manual of psychiatry and law. Arlington, VA: American Psychiatric Publishing, Inc; 2007.
10. American Psychiatric Association. Principles of medical ethics with annotations especially applicable to psychiatry. Section 4, annotation 8. Washington, DC: American Psychiatric Publishing, Inc; 2001.
11. Simon RI, Gutheil TG. Sudden improvement in high-risk suicidal patients: should it be trusted? Psych Serv. 2009; 60(3):387-389.
12. Pokorny AD. Prediction of suicide in psychiatric patients. Report of a prospective study. Arch Gen Psychiatry. 1983; 4(3):249-257.
13. Simon RI. Imminent suicide: the illusion of short-term prediction. Suicide Life Threat Behav. 2006;36(3): 296-301.
14. Simon RI, Shuman DW. Therapeutic risk management of clinical-legal dilemmas: should it be a core competency? J Am Acad Psychiatry Law. 2009;37(2):155-161.
15. Linehan MM, Goodstein JL, Nielsen SL, et al. Reasons for staying alive when you are thinking of killing yourself: the reasons for living inventory. J Consult Clin Psychol. 1983;51(2):276-286.
16. Malone KM, Oquendo MA, Hass GL, et al. Protective factors against suicidal acts in major depression: reasons for living. Am J Psychiatry. 2000;157(7):1084-1088.
17. Baldessarini RJ, Pompili M, Tondo L. Bipolar disorder. In: Simon RI, Hales RE, eds. Textbook of suicide assessment and management. Arlington, VA: American Psychiatric Publishing, Inc; 2006:159-176.

References


1. Stanford EJ, Goetz RR, Bloom JD. The No Harm Contract in the emergency assessment of suicide risk. J Clin Psychiatry. 1994;55(8):344-348.
2. Simon RI, Hales RE, eds. Textbook of suicide assessment and management. 2nd ed. Arlington, VA: American Psychiatric Publishing, Inc.; 2012.
3. Practice guidelines for the assessment and treatment of patients with suicidal behaviors [Erratum in Am J Psychiatry. 2004;161(4):776]. Am J Psychiatry. 2003;160(suppl 11):1-60.
4. Simon RI. Suicide risk assessment forms: form over substance? J Am Acad Psychiatry Law. 2009;37(3): 290-293.
5. Resnick PJ. Recognizing that the suicidal patient views you as an ‘adversary.’ Current Psychiatry. 2002;1(1):8.
6. Dervic K, Oquendo MA, Grunebaum MF, et al. Religious affiliation and suicide attempt. Am J Psychiatry. 2004; 161(12):2303-2308.
7. Harris CE, Barraclough B. Suicide as an outcome for mental disorders. A meta-analysis. Br J Psychiatry. 1997;170:205-228.
8. Health insurance portability and accountability act of 1996. Pub L No. 104-191.
9. Simon RI, Shuman DW. Clinical manual of psychiatry and law. Arlington, VA: American Psychiatric Publishing, Inc; 2007.
10. American Psychiatric Association. Principles of medical ethics with annotations especially applicable to psychiatry. Section 4, annotation 8. Washington, DC: American Psychiatric Publishing, Inc; 2001.
11. Simon RI, Gutheil TG. Sudden improvement in high-risk suicidal patients: should it be trusted? Psych Serv. 2009; 60(3):387-389.
12. Pokorny AD. Prediction of suicide in psychiatric patients. Report of a prospective study. Arch Gen Psychiatry. 1983; 4(3):249-257.
13. Simon RI. Imminent suicide: the illusion of short-term prediction. Suicide Life Threat Behav. 2006;36(3): 296-301.
14. Simon RI, Shuman DW. Therapeutic risk management of clinical-legal dilemmas: should it be a core competency? J Am Acad Psychiatry Law. 2009;37(2):155-161.
15. Linehan MM, Goodstein JL, Nielsen SL, et al. Reasons for staying alive when you are thinking of killing yourself: the reasons for living inventory. J Consult Clin Psychol. 1983;51(2):276-286.
16. Malone KM, Oquendo MA, Hass GL, et al. Protective factors against suicidal acts in major depression: reasons for living. Am J Psychiatry. 2000;157(7):1084-1088.
17. Baldessarini RJ, Pompili M, Tondo L. Bipolar disorder. In: Simon RI, Hales RE, eds. Textbook of suicide assessment and management. Arlington, VA: American Psychiatric Publishing, Inc; 2006:159-176.

Issue
Current Psychiatry - 13(10)
Issue
Current Psychiatry - 13(10)
Page Number
26-28, 30-32
Page Number
26-28, 30-32
Publications
Publications
Topics
Article Type
Display Headline
How do you score on this self-assessment of suicide risk management?: First of 2 parts
Display Headline
How do you score on this self-assessment of suicide risk management?: First of 2 parts
Legacy Keywords
suicide, suicide risk, suicide assessment, suicide risk assessment
Legacy Keywords
suicide, suicide risk, suicide assessment, suicide risk assessment
Sections
Article Source

PURLs Copyright

Inside the Article

Article PDF Media

Passive suicidal ideation: Still a high-risk clinical scenario

Article Type
Changed
Mon, 07/13/2020 - 09:16
Display Headline
Passive suicidal ideation: Still a high-risk clinical scenario

The commonly held belief that passive suicidal ide­ation poses less risk for suicide than active suicidal ideation is steeped in the lore of psychiatric practice. “Passive suicidal ideation” appears countless times in psychiatric records, articles, texts, guidelines, and clinical discourse. When a patient reports passive suicide ideation, the clinician may seize upon it as an indicator of low risk of suicide. The clinician may feel relieved and not perform a thorough suicide risk assessment.

Whether suicide ideation is active or passive, the goal is the same—terminating one’s life. Suicidal ideation, such as the wish to die during sleep, to be killed in an accident, or to develop terminal cancer, may seem relatively innocu­ous, but it can be just as ominous as thoughts of hanging oneself. Although passive suicidal ideation may allow time for interventions, passive ideation can suddenly turn active.


CASE "I love my family too much to hurt myself"

Mr. F, a 52-year-old business executive, is brought to the hospital emergency room by his wife. His business is heading to bankruptcy and he is unable to go to the office and face his employees. Mr. F cannot sleep or eat, spending most of the day on the couch crying. His wife has threatened her husband with separation if he does not seek psychiatric treatment.

Mr. F tells the emergency room psychiatrist, “I am stressed but have no intention of hurting myself. I love my wife and kids too much to put them through that.” He admits to having wishes to die during sleep, but reports, “I can’t sleep anyway.” His wife finds a loaded gun in the glove compartment of his car, but he says the “gun is for my protection.” He angrily denies any suicidal ideation and protests, “I do not need to be here.” His wife insists that he be treated, stating, “I will not take my husband home in his condition.”

Mr. F refuses psychiatric hospitalization but changes his mind when confronted with the alternative of involuntary hospitalization. He admits that, unknown to his wife, he recently purchased a $2 million life insurance policy and made funeral arrangements. He planned to kill himself with his revolver. A thorough suicide risk assessment reveals a number of evidence-based risk factors that place the patient at acute, high risk for suicide.


Passive ideation is active

When a patient reports passive suicidal ideation, active suicidal ideation invariably is present. No bright line separates them. Suicidal ideation, active or passive, contains a dynamic mix of ambivalent thoughts and feelings along a continuum of severity. It reflects ongoing change in the patient’s psychiatric disorder.1

Reynolds et al2 assessed the clinical correlates of active suicidal ideation vs passive death wishes in geriatric patients with recurrent major depression. Their data challenged the utility of distinguishing active and passive suicidal ideation. The authors also noted that the patient’s ideation can change from passive to active during an episode of illness. They recommended that clinicians be no less vigilant with patients expressing passive suicidal ideation.

Suicidal ideation that expresses active or passive methods of suicide usually reflects psychodynamic, cultural, religious, and moral issues as well as evasiveness, guardedness, denial, and other factors. Assessing passive suicidal ideation may reveal few protective factors, which may increase the patient’s suicide risk.

Patients often find it easier to talk about protective factors than suicidal thoughts. Patients whose culture or religion strongly condemns suicide may feel less conflicted reporting suicide ideation in the passive mode, if at all. For the patient who is determined to commit suicide, passive expression of suicide intent may indicate minimizing risk or deception, as seen in the case described here.3

“Fleeting” suicidal ideation, a frequent companion of “passive” suicidal ideation, also requires careful evaluation. In a study of 100 patients who made severe suicide attempts, Hall et al4 found that 69 reported only fleeting or no suicidal ideation before their attempt. “Fleeting” thoughts of suicide must not be accepted at face value but require thorough assessment.

Structured assessments instruments for evaluating suicide ideation are available. The Chronological Assessment of Suicidal Events (case approach) is designed to uncover detailed information related to the patient’s suicidal ideation.5 The Scale for Suicide Ideation, and the later version, Beck Scale for Suicide Ideation,6 rates passive suicidal ideation on a 3-point Likerttype scale as:
     0 “would take precautions to save life”
     1 “would leave life/death to chance (eg, carelessly crossing a busy street)”
     2 “would avoid steps necessary to save or maintain life (eg, diabetic ceasing to take insulin).”

Although the Beck scales have psychometric properties (reliability and validity), no scale can substitute for thorough clinical assessment of suicidal ideation. If used, ratings scales or checklists of suicidal ideation can alert clinicians to thoroughly assess this crucial symptom of suicide risk.

 

 

When treating a suicidal patient, clinicians often experience complex, distressing feelings. Maltzberger and Buie7 describe anger, frustration, despair, and even hate toward the suicidal patient. In addition to the devastating loss of one’s patient, fears of a lawsuits and damage to one’s professional competence and reputation may arise if the patient attempts or completes suicide. These can all lead a clinician to prematurely accept a patient’s statement regarding passive suicidal ideation rather than conduct a thorough suicide risk assessment. Consultation should be considered.


The necessity of action

Suicidal ideation must be carefully assessed—not labeled. Passive suicidal ideation should not deter a clinician from performing a thorough suicide risk assessment. A patient’s report of passive suicidal ideation is not an end but a beginning of thorough suicide risk assessment.


Bottom Line

Passive suicidal ideation, such as a wish to die during sleep or being killed in an accident, does not indicate that a patient is at a low risk of suicide. A thorough suicide risk assessment may reveal active suicidal ideation that informs treatment and management interventions.


Related Resources
• Simon RI. Suicide rehearsals: A high risk psychiatric emer­gency. Current Psychiatry. 2012;11(7):28-32.

• Baca-Garcia E, Perez-Rodriguez MM, Oquendo MA, et al. Estimating risk for suicide attempt: are we asking the right questions? Passive suicidal ideation as a marker for suicidal behavior. J Affect Disord. 2011;134(1-3):327-332.

References


1. Isometsä ET, Lönnqvist JK. Suicide attempts preceding completed suicide. Br J Psychiatry. 1998;173:531-535.
2. Szanto K, Reynolds CF, Frank E, et al. Suicide in elderly depressed patients: is active vs. passive suicidal ideation a clinically valid distinction? Am J Geriatr Psychiatry. 1996;4(3):197-207.
3. Simon RI. Behavioral risk assessment of the guarded suicidal patient. Suicide Life Threat Behav. 2008;38(5):517-522.
4. Hall RC, Platt DE, Hall RC. Suicide risk assessment: a review of risk factors for suicide in 100 patients who made severe suicide attempts. Evaluations of suicide risk in a time of managed care. Psychosomatics. 1999;40(1):18-27.
5. Shea SC. The interpersonal art of suicide assessment interviewing techniques for uncovering suicidal intent, ideation and actions. In: Simon RI, Hale RE, eds. American Psychiatric Publishing textbook of suicide assessment and management. Arlington, VA: American Psychiatric Publishing; 2012:29-56.
6. Rush AJ Jr, First MB, Blacker D. Suicide risk measures. In: Rush AJ Jr, First MB, Blacker D, eds. Handbook of psychiatric measures, 2nd ed. Arlington, VA: American Psychiatric Publishing; 2008:242-244.
7. Maltzberger JT, Buie DH. Countertransference hate in the treatment of suicidal patients. Arch Gen Psychiatry. 1974; 30(5):625-633.
8. Simon RI. Suicide risk assessment: gateway to treatment and management. In: Simon RI, Hale RE, eds. American Psychiatric Publishing textbook of suicide assessment and management. Arlington, VA: American Psychiatric Publishing; 2012:3-28.

Article PDF
Author and Disclosure Information

Robert I. Simon, MD
Clinical Professor of Psychiatry, Georgetown University School of Medicine, Washington, DC
Chairman, Department of Psychiatry, Suburban Hospital, a member of Johns Hopkins Medicine,Bethesda, Maryland.

DisclosureDr. Simon reports no financial relationships with any company whose products are mentioned in this article or with manufacturers of competing products.

 

Adapted with permission from: Simon RI: Preventing patient suicide: Clinical assessment and management. Arlington, VA: American Psychiatric Publishing, Inc.; 2011.

Issue
Current Psychiatry - 13(3)
Publications
Topics
Page Number
13-15
Legacy Keywords
suicide, passive ideation, suicide risk assessment
Sections
Author and Disclosure Information

Robert I. Simon, MD
Clinical Professor of Psychiatry, Georgetown University School of Medicine, Washington, DC
Chairman, Department of Psychiatry, Suburban Hospital, a member of Johns Hopkins Medicine,Bethesda, Maryland.

DisclosureDr. Simon reports no financial relationships with any company whose products are mentioned in this article or with manufacturers of competing products.

 

Adapted with permission from: Simon RI: Preventing patient suicide: Clinical assessment and management. Arlington, VA: American Psychiatric Publishing, Inc.; 2011.

Author and Disclosure Information

Robert I. Simon, MD
Clinical Professor of Psychiatry, Georgetown University School of Medicine, Washington, DC
Chairman, Department of Psychiatry, Suburban Hospital, a member of Johns Hopkins Medicine,Bethesda, Maryland.

DisclosureDr. Simon reports no financial relationships with any company whose products are mentioned in this article or with manufacturers of competing products.

 

Adapted with permission from: Simon RI: Preventing patient suicide: Clinical assessment and management. Arlington, VA: American Psychiatric Publishing, Inc.; 2011.

Article PDF
Article PDF
Related Articles

The commonly held belief that passive suicidal ide­ation poses less risk for suicide than active suicidal ideation is steeped in the lore of psychiatric practice. “Passive suicidal ideation” appears countless times in psychiatric records, articles, texts, guidelines, and clinical discourse. When a patient reports passive suicide ideation, the clinician may seize upon it as an indicator of low risk of suicide. The clinician may feel relieved and not perform a thorough suicide risk assessment.

Whether suicide ideation is active or passive, the goal is the same—terminating one’s life. Suicidal ideation, such as the wish to die during sleep, to be killed in an accident, or to develop terminal cancer, may seem relatively innocu­ous, but it can be just as ominous as thoughts of hanging oneself. Although passive suicidal ideation may allow time for interventions, passive ideation can suddenly turn active.


CASE "I love my family too much to hurt myself"

Mr. F, a 52-year-old business executive, is brought to the hospital emergency room by his wife. His business is heading to bankruptcy and he is unable to go to the office and face his employees. Mr. F cannot sleep or eat, spending most of the day on the couch crying. His wife has threatened her husband with separation if he does not seek psychiatric treatment.

Mr. F tells the emergency room psychiatrist, “I am stressed but have no intention of hurting myself. I love my wife and kids too much to put them through that.” He admits to having wishes to die during sleep, but reports, “I can’t sleep anyway.” His wife finds a loaded gun in the glove compartment of his car, but he says the “gun is for my protection.” He angrily denies any suicidal ideation and protests, “I do not need to be here.” His wife insists that he be treated, stating, “I will not take my husband home in his condition.”

Mr. F refuses psychiatric hospitalization but changes his mind when confronted with the alternative of involuntary hospitalization. He admits that, unknown to his wife, he recently purchased a $2 million life insurance policy and made funeral arrangements. He planned to kill himself with his revolver. A thorough suicide risk assessment reveals a number of evidence-based risk factors that place the patient at acute, high risk for suicide.


Passive ideation is active

When a patient reports passive suicidal ideation, active suicidal ideation invariably is present. No bright line separates them. Suicidal ideation, active or passive, contains a dynamic mix of ambivalent thoughts and feelings along a continuum of severity. It reflects ongoing change in the patient’s psychiatric disorder.1

Reynolds et al2 assessed the clinical correlates of active suicidal ideation vs passive death wishes in geriatric patients with recurrent major depression. Their data challenged the utility of distinguishing active and passive suicidal ideation. The authors also noted that the patient’s ideation can change from passive to active during an episode of illness. They recommended that clinicians be no less vigilant with patients expressing passive suicidal ideation.

Suicidal ideation that expresses active or passive methods of suicide usually reflects psychodynamic, cultural, religious, and moral issues as well as evasiveness, guardedness, denial, and other factors. Assessing passive suicidal ideation may reveal few protective factors, which may increase the patient’s suicide risk.

Patients often find it easier to talk about protective factors than suicidal thoughts. Patients whose culture or religion strongly condemns suicide may feel less conflicted reporting suicide ideation in the passive mode, if at all. For the patient who is determined to commit suicide, passive expression of suicide intent may indicate minimizing risk or deception, as seen in the case described here.3

“Fleeting” suicidal ideation, a frequent companion of “passive” suicidal ideation, also requires careful evaluation. In a study of 100 patients who made severe suicide attempts, Hall et al4 found that 69 reported only fleeting or no suicidal ideation before their attempt. “Fleeting” thoughts of suicide must not be accepted at face value but require thorough assessment.

Structured assessments instruments for evaluating suicide ideation are available. The Chronological Assessment of Suicidal Events (case approach) is designed to uncover detailed information related to the patient’s suicidal ideation.5 The Scale for Suicide Ideation, and the later version, Beck Scale for Suicide Ideation,6 rates passive suicidal ideation on a 3-point Likerttype scale as:
     0 “would take precautions to save life”
     1 “would leave life/death to chance (eg, carelessly crossing a busy street)”
     2 “would avoid steps necessary to save or maintain life (eg, diabetic ceasing to take insulin).”

Although the Beck scales have psychometric properties (reliability and validity), no scale can substitute for thorough clinical assessment of suicidal ideation. If used, ratings scales or checklists of suicidal ideation can alert clinicians to thoroughly assess this crucial symptom of suicide risk.

 

 

When treating a suicidal patient, clinicians often experience complex, distressing feelings. Maltzberger and Buie7 describe anger, frustration, despair, and even hate toward the suicidal patient. In addition to the devastating loss of one’s patient, fears of a lawsuits and damage to one’s professional competence and reputation may arise if the patient attempts or completes suicide. These can all lead a clinician to prematurely accept a patient’s statement regarding passive suicidal ideation rather than conduct a thorough suicide risk assessment. Consultation should be considered.


The necessity of action

Suicidal ideation must be carefully assessed—not labeled. Passive suicidal ideation should not deter a clinician from performing a thorough suicide risk assessment. A patient’s report of passive suicidal ideation is not an end but a beginning of thorough suicide risk assessment.


Bottom Line

Passive suicidal ideation, such as a wish to die during sleep or being killed in an accident, does not indicate that a patient is at a low risk of suicide. A thorough suicide risk assessment may reveal active suicidal ideation that informs treatment and management interventions.


Related Resources
• Simon RI. Suicide rehearsals: A high risk psychiatric emer­gency. Current Psychiatry. 2012;11(7):28-32.

• Baca-Garcia E, Perez-Rodriguez MM, Oquendo MA, et al. Estimating risk for suicide attempt: are we asking the right questions? Passive suicidal ideation as a marker for suicidal behavior. J Affect Disord. 2011;134(1-3):327-332.

The commonly held belief that passive suicidal ide­ation poses less risk for suicide than active suicidal ideation is steeped in the lore of psychiatric practice. “Passive suicidal ideation” appears countless times in psychiatric records, articles, texts, guidelines, and clinical discourse. When a patient reports passive suicide ideation, the clinician may seize upon it as an indicator of low risk of suicide. The clinician may feel relieved and not perform a thorough suicide risk assessment.

Whether suicide ideation is active or passive, the goal is the same—terminating one’s life. Suicidal ideation, such as the wish to die during sleep, to be killed in an accident, or to develop terminal cancer, may seem relatively innocu­ous, but it can be just as ominous as thoughts of hanging oneself. Although passive suicidal ideation may allow time for interventions, passive ideation can suddenly turn active.


CASE "I love my family too much to hurt myself"

Mr. F, a 52-year-old business executive, is brought to the hospital emergency room by his wife. His business is heading to bankruptcy and he is unable to go to the office and face his employees. Mr. F cannot sleep or eat, spending most of the day on the couch crying. His wife has threatened her husband with separation if he does not seek psychiatric treatment.

Mr. F tells the emergency room psychiatrist, “I am stressed but have no intention of hurting myself. I love my wife and kids too much to put them through that.” He admits to having wishes to die during sleep, but reports, “I can’t sleep anyway.” His wife finds a loaded gun in the glove compartment of his car, but he says the “gun is for my protection.” He angrily denies any suicidal ideation and protests, “I do not need to be here.” His wife insists that he be treated, stating, “I will not take my husband home in his condition.”

Mr. F refuses psychiatric hospitalization but changes his mind when confronted with the alternative of involuntary hospitalization. He admits that, unknown to his wife, he recently purchased a $2 million life insurance policy and made funeral arrangements. He planned to kill himself with his revolver. A thorough suicide risk assessment reveals a number of evidence-based risk factors that place the patient at acute, high risk for suicide.


Passive ideation is active

When a patient reports passive suicidal ideation, active suicidal ideation invariably is present. No bright line separates them. Suicidal ideation, active or passive, contains a dynamic mix of ambivalent thoughts and feelings along a continuum of severity. It reflects ongoing change in the patient’s psychiatric disorder.1

Reynolds et al2 assessed the clinical correlates of active suicidal ideation vs passive death wishes in geriatric patients with recurrent major depression. Their data challenged the utility of distinguishing active and passive suicidal ideation. The authors also noted that the patient’s ideation can change from passive to active during an episode of illness. They recommended that clinicians be no less vigilant with patients expressing passive suicidal ideation.

Suicidal ideation that expresses active or passive methods of suicide usually reflects psychodynamic, cultural, religious, and moral issues as well as evasiveness, guardedness, denial, and other factors. Assessing passive suicidal ideation may reveal few protective factors, which may increase the patient’s suicide risk.

Patients often find it easier to talk about protective factors than suicidal thoughts. Patients whose culture or religion strongly condemns suicide may feel less conflicted reporting suicide ideation in the passive mode, if at all. For the patient who is determined to commit suicide, passive expression of suicide intent may indicate minimizing risk or deception, as seen in the case described here.3

“Fleeting” suicidal ideation, a frequent companion of “passive” suicidal ideation, also requires careful evaluation. In a study of 100 patients who made severe suicide attempts, Hall et al4 found that 69 reported only fleeting or no suicidal ideation before their attempt. “Fleeting” thoughts of suicide must not be accepted at face value but require thorough assessment.

Structured assessments instruments for evaluating suicide ideation are available. The Chronological Assessment of Suicidal Events (case approach) is designed to uncover detailed information related to the patient’s suicidal ideation.5 The Scale for Suicide Ideation, and the later version, Beck Scale for Suicide Ideation,6 rates passive suicidal ideation on a 3-point Likerttype scale as:
     0 “would take precautions to save life”
     1 “would leave life/death to chance (eg, carelessly crossing a busy street)”
     2 “would avoid steps necessary to save or maintain life (eg, diabetic ceasing to take insulin).”

Although the Beck scales have psychometric properties (reliability and validity), no scale can substitute for thorough clinical assessment of suicidal ideation. If used, ratings scales or checklists of suicidal ideation can alert clinicians to thoroughly assess this crucial symptom of suicide risk.

 

 

When treating a suicidal patient, clinicians often experience complex, distressing feelings. Maltzberger and Buie7 describe anger, frustration, despair, and even hate toward the suicidal patient. In addition to the devastating loss of one’s patient, fears of a lawsuits and damage to one’s professional competence and reputation may arise if the patient attempts or completes suicide. These can all lead a clinician to prematurely accept a patient’s statement regarding passive suicidal ideation rather than conduct a thorough suicide risk assessment. Consultation should be considered.


The necessity of action

Suicidal ideation must be carefully assessed—not labeled. Passive suicidal ideation should not deter a clinician from performing a thorough suicide risk assessment. A patient’s report of passive suicidal ideation is not an end but a beginning of thorough suicide risk assessment.


Bottom Line

Passive suicidal ideation, such as a wish to die during sleep or being killed in an accident, does not indicate that a patient is at a low risk of suicide. A thorough suicide risk assessment may reveal active suicidal ideation that informs treatment and management interventions.


Related Resources
• Simon RI. Suicide rehearsals: A high risk psychiatric emer­gency. Current Psychiatry. 2012;11(7):28-32.

• Baca-Garcia E, Perez-Rodriguez MM, Oquendo MA, et al. Estimating risk for suicide attempt: are we asking the right questions? Passive suicidal ideation as a marker for suicidal behavior. J Affect Disord. 2011;134(1-3):327-332.

References


1. Isometsä ET, Lönnqvist JK. Suicide attempts preceding completed suicide. Br J Psychiatry. 1998;173:531-535.
2. Szanto K, Reynolds CF, Frank E, et al. Suicide in elderly depressed patients: is active vs. passive suicidal ideation a clinically valid distinction? Am J Geriatr Psychiatry. 1996;4(3):197-207.
3. Simon RI. Behavioral risk assessment of the guarded suicidal patient. Suicide Life Threat Behav. 2008;38(5):517-522.
4. Hall RC, Platt DE, Hall RC. Suicide risk assessment: a review of risk factors for suicide in 100 patients who made severe suicide attempts. Evaluations of suicide risk in a time of managed care. Psychosomatics. 1999;40(1):18-27.
5. Shea SC. The interpersonal art of suicide assessment interviewing techniques for uncovering suicidal intent, ideation and actions. In: Simon RI, Hale RE, eds. American Psychiatric Publishing textbook of suicide assessment and management. Arlington, VA: American Psychiatric Publishing; 2012:29-56.
6. Rush AJ Jr, First MB, Blacker D. Suicide risk measures. In: Rush AJ Jr, First MB, Blacker D, eds. Handbook of psychiatric measures, 2nd ed. Arlington, VA: American Psychiatric Publishing; 2008:242-244.
7. Maltzberger JT, Buie DH. Countertransference hate in the treatment of suicidal patients. Arch Gen Psychiatry. 1974; 30(5):625-633.
8. Simon RI. Suicide risk assessment: gateway to treatment and management. In: Simon RI, Hale RE, eds. American Psychiatric Publishing textbook of suicide assessment and management. Arlington, VA: American Psychiatric Publishing; 2012:3-28.

References


1. Isometsä ET, Lönnqvist JK. Suicide attempts preceding completed suicide. Br J Psychiatry. 1998;173:531-535.
2. Szanto K, Reynolds CF, Frank E, et al. Suicide in elderly depressed patients: is active vs. passive suicidal ideation a clinically valid distinction? Am J Geriatr Psychiatry. 1996;4(3):197-207.
3. Simon RI. Behavioral risk assessment of the guarded suicidal patient. Suicide Life Threat Behav. 2008;38(5):517-522.
4. Hall RC, Platt DE, Hall RC. Suicide risk assessment: a review of risk factors for suicide in 100 patients who made severe suicide attempts. Evaluations of suicide risk in a time of managed care. Psychosomatics. 1999;40(1):18-27.
5. Shea SC. The interpersonal art of suicide assessment interviewing techniques for uncovering suicidal intent, ideation and actions. In: Simon RI, Hale RE, eds. American Psychiatric Publishing textbook of suicide assessment and management. Arlington, VA: American Psychiatric Publishing; 2012:29-56.
6. Rush AJ Jr, First MB, Blacker D. Suicide risk measures. In: Rush AJ Jr, First MB, Blacker D, eds. Handbook of psychiatric measures, 2nd ed. Arlington, VA: American Psychiatric Publishing; 2008:242-244.
7. Maltzberger JT, Buie DH. Countertransference hate in the treatment of suicidal patients. Arch Gen Psychiatry. 1974; 30(5):625-633.
8. Simon RI. Suicide risk assessment: gateway to treatment and management. In: Simon RI, Hale RE, eds. American Psychiatric Publishing textbook of suicide assessment and management. Arlington, VA: American Psychiatric Publishing; 2012:3-28.

Issue
Current Psychiatry - 13(3)
Issue
Current Psychiatry - 13(3)
Page Number
13-15
Page Number
13-15
Publications
Publications
Topics
Article Type
Display Headline
Passive suicidal ideation: Still a high-risk clinical scenario
Display Headline
Passive suicidal ideation: Still a high-risk clinical scenario
Legacy Keywords
suicide, passive ideation, suicide risk assessment
Legacy Keywords
suicide, passive ideation, suicide risk assessment
Sections
Disallow All Ads
Alternative CME
Use ProPublica
Hide sidebar & use full width
render the right sidebar.
Conference Recap Checkbox
Not Conference Recap
Clinical Edge
Display the Slideshow in this Article
Article PDF Media

Suicide rehearsals: A high-risk psychiatric emergency

Article Type
Changed
Mon, 07/13/2020 - 09:16
Display Headline
Suicide rehearsals: A high-risk psychiatric emergency

A suicide rehearsal is a behavioral enactment of a suicide method, usually as part of a suicide plan. A mental suicide rehearsal is a process that evolves over time into a plan. Patients who are intent on attempting suicide usually do not reveal their plans. However, behavioral rehearsals display specific clinical characteristics that speak louder than the guarded patient’s denials, revealing the patient’s suicide plan (Table).

Suicide rehearsals may precede suicide attempts or suicide completions. The percentage of patients who stage suicide rehearsals before attempting or completing suicide is unknown; however, in my experience, suicide rehearsals are relatively common. This article describes suicide rehearsals, and offers 4 cases that illustrate what clinicians can learn from rehearsals to improve their patients’ safety.

Table

Clinical characteristics of suicide rehearsals

 

Guarded patient
Behavioral enactment of a suicide method
Lethal means
Presumptive acute, high risk of suicide
Severe mental illness
Suicide attempt often within hours or days
Rehearsal usually covert
Rehearsal event or multiple events

The psychology behind suicide rehearsals

Rehearsing suicidal behavior can lower the barrier to a suicide plan, thereby increasing a patient’s resolve and risk. Joiner1 notes that engaging in behavioral or mental suicide rehearsals increases the risk of suicide. Moreover, rehearsals diminish the prohibition against suicidal behavior and the fear of pain and dying. Examples of rehearsal psychology include:

 

  • overcoming ambivalence about dying
  • desensitizing anxiety about performing the suicide act
  • testing or “perfecting” the method of a planned suicide
  • firming one’s resolve to complete suicide.

Other non-lethal motivations include “a cry for help” and self-injurious behaviors motivated by external gains. Patients who do not intend to attempt suicide may openly rehearse low-risk methods, such as superficial cutting.

Rehearsal characteristics

 

Suicide rehearsals can be confused with aborted, interrupted, or failed suicide attempts. Suicide rehearsals usually are associated with severe psychiatric illness and high-risk lethal methods of attempting suicide. My experience is that suicide attempts or suicide completions often follow a rehearsal within a few hours or days. However, no short-term suicide risk factors—within hours, days, or weeks—can predict when or if a rehearsed suicide will proceed to a suicide attempt.2

A suicide rehearsal is presumptive evidence that the patient is at acute, high risk for suicide and immediate clinical intervention is necessary. A rehearsal allows the clinician to explore the various methods of suicide that the patient has considered, including prior rehearsals. Knowledge of prior rehearsals can inform the clinician’s management of the current suicide rehearsal.

Suicide rehearsals often are conducted covertly. On inpatient psychiatric units, the rehearsal usually is discovered by staff members or reported by other patients. In outpatient settings, the patient or a significant other may report a rehearsal.

The suicide method displayed in a rehearsal may change. A patient who is rehearsing a hanging may attempt suicide by overdose or a firearm. In a systematic review of prior suicide attempts (N = 1,397), Isometsä et al3 found that 82% of patients used 2 or more different methods in suicide attempts, including the completed suicide. However, in a cohort study of 48,649 individuals admitted to a hospital after an attempted suicide, Runeson et al4 found that patients who attempt suicide often used the same method in completed suicide (ie, >90% by hanging for both men and women). Therefore, when taking measures to restrict the patient’s access to lethal means, safety efforts should not be limited to the method used in the suicide rehearsal. Patients can always substitute methods.

Making overall preparations for suicide—for example, making a will, giving away valuable possessions, or putting financial affairs in order—could be confused with a suicide rehearsal, which displays the lethal method to be used in a suicide attempt, often after preparations are made. Suicide rehearsals tend to occur much closer in time to the suicide attempt than preparations for suicide. Similarly, a patient’s plan to hoard drugs for a suicide attempt is not the same as ingesting a sub-lethal dose of a drug to test his or her resolve to die.

By definition, impulsive suicide attempts are not rehearsed. However, an individual’s suicide rehearsal can impulsively segue into a suicide attempt. In a case control study (N = 153) Simon et al5 found that 24% of patients spent 6 found that 26% of individuals with lifetime suicide ideation transitioned from suicide ideation to an unplanned suicide attempt. In my experience, a suicide rehearsal before a suicide completion is presumptive evidence against an impulsive suicide.

 

 

Patients contemplating suicide may visit Web sites with instructions on “how to suicide,” providing “virtual” opportunities to rehearse suicide.7 Patients who are at risk for suicide should be asked if they have searched the Internet for suicide methodology.

What we can learn from rehearsals

Although the following case examples are fictional, they illustrate suicide rehearsals encountered in my clinical and forensic practice.

CASE 1: Looking for a location

Ms. B, a 28-year-old divorced mother of 2, is observed tarrying at the high point of a bridge on successive days. When police arrive and question her, she becomes agitated and distraught. Ms. B admits to “scoping out” the bridge and is taken to a hospital emergency room (ER). In the ER, Ms. B discloses, “I was looking for a good spot to jump.” She tells the triage nurse that she is very depressed but, “I couldn’t do it to my children.” Ms. B is placed in an unlocked room while she waits to be assessed by a psychiatrist. She leaves the ER, runs to a nearby parking garage, and jumps from the top level to her death.

Comment: A patient’s denial of suicide intent following a suicide rehearsal cannot be relied upon. Ms. B’s rehearsal revealed a plan with high-risk suicide intent and a lethal suicide method. Systematic suicide assessment that informs immediate clinical intervention is required.

CASE 2: Changing lethal means

Mr. N, a 43-year-old chief executive officer of a large company, is observed by an assistant loading and unloading a revolver at his desk. Alarmed, the assistant calls the company physician. Mr. N refuses psychiatric treatment, saying, “I’ll be all right; this is just a passing thing.” His wife tells the physician that her husband has a history of bipolar disorder but no prior suicide attempts. Guns and ammunition are removed from the home. One week later, Mr. N is found hanging in his garage. A loaded pistol is discovered in the glove compartment of his car.

Comment: There is no certainty that a subsequent suicide attempt will replicate the rehearsed method. A psychological autopsy was conducted, but no explanation was found for why Mr. N chose hanging after having rehearsed suicide with a loaded handgun. His wife thought that her husband, a very tidy person, did not want to leave a mess.

CASE 3: Grieving and depressed

 

Mr. O, age 67, is depressed after recently losing his wife. He considers a number of suicide methods. Mr. O decides to use a plastic bag to suffocate himself because he believes that this method will allow him to change his mind. Mr. O practices tying the bag tight around his neck. During this rehearsal, he realizes that he does not want to die. Instead, he pursues grief counseling.

 

Comment: For some patients, the act of rehearsing suicide can help them resolve ambivalent feelings about wanting to die in favor of wanting to live.

 

CASE 4: Suicide method and the Internet

Ms. S, a 22-year-old college student, is undergoing outpatient treatment for depression. She is accumulating prescription drugs to take as an overdose. Ms. S also searches the Internet for information about other suicide methods. Because she wants a “sure” method of suicide, she persuades an acquaintance to purchase a handgun. In private, Ms. S places the unloaded gun to her head and plays “Russian roulette,” pulling the trigger several times. Her mother discovers the gun and confronts her daughter. Ms. S is hospitalized on a closed psychiatric unit and tells a staff member, “I was practicing suicide with the gun.” Before Ms. S is discharged from the hospital, her parents are advised to watch for suicidal behaviors, especially the recurrence of rehearsals that indicate an acute, high suicide risk. Ms. S’s Internet use is restricted and monitored.

Comment: Suicide rehearsal with a gun reinforces the belief that a firearm death is quick and easy.8 Reaching for a loaded gun takes less time than most other methods of suicide. Patients who rehearse suicide with a gun should be prevented from having access to any firearms, weapons, or other highly lethal means of suicide.

Recognition and intervention

A guarded psychiatric inpatient who is intent on attempting suicide is unmasked when the discovery of a suicidal rehearsal reveals a suicide plan. This creates an opportunity for clinicians to intervene. The patient may attempt to cover up suicidal intent by stating, “I was just playing around” or “I just wanted to get attention.” Recognizing the emergency posed by a suicide rehearsal informs treatment. Safety measures—including 1-to-1 supervision—may be necessary during a period of acute, high suicide risk. The patient’s diagnosis, severity of illness, and treatment require reevaluation.

 

 

 

An outpatient who performs a suicide rehearsal should be considered at acute, high risk for suicide, and immediate psychiatric hospitalization may be necessary. Whether as an inpatient or outpatient, the patient’s suicide intent and plan require careful exploration. The information gained will guide treatment and management decisions. Continuing systematic suicide risk assessment is essential.

Related Resources

 

  • American Association of Suicidology. www.suicidology.org.
  • Joiner T. Why people die by suicide. Cambridge, MA: Harvard University Press; 2007.
  • American Psychiatric Association. Practice guideline for the assessment and treatment of patients with suicidal behaviors. Washington, DC: American Psychiatric Publishing, Inc. 2003.
  • Simon RI. Preventing patient suicide: clinical assessment and management. Arlington, VA: American Psychiatric Publishing, Inc.; 2011.

Disclosure

Dr. Simon reports no financial relationship with any company whose products are mentioned in this article or with manufacturers of competing products.

References

 

1. Joiner TE, Jr. The trajectory of suicidal behavior over time. Suicide Life Threat Behav. 2002;32(1):33-41.

2. Simon RI. Imminent suicide: the illusion of short-term prediction. Suicide Life Threat Behav. 2006;36(3):296-301.

3. Isometsä ET, Lönnqvist JK. Suicide attempts preceding completed suicide. Br J Psychiatry. 1998;173:531-535.

4. Runeson B, Tidemalm D, Dahlin M, et al. Method of attempted suicide as a predictor of subsequent successful suicide: national long-term cohort study. BMJ. 2010;341:c3222. doi:10.1136/bmj.63222.

5. Simon TR, Swann AC, Powell KE, et al. Characteristics of impulsive suicide attempts and attempters. Suicide Life Threat Behav. 2001;32(suppl):49-59.

6. Kessler RC, Borges G, Walters EE. Prevalence of and risk factors for lifetime suicide attempts in the National Comorbidity Study. Arch Gen Psychiatry. 1999;56(7):617-626.

7. Recupero PR. Suicide and the Internet. In: Simon RI Hales RE, eds. The American Psychiatric Publishing textbook of suicide assessment and management. 2nd ed. Arlington, VA: American Psychiatric Publishing, Inc.; 2012:515–538.

8. Simon RI. Gun safety management with patients at risk for suicide. Suicide Life Threat Behav. 2007;37(5):518-526.

Article PDF
Author and Disclosure Information

 

Robert I. Simon, MD
Clinical Professor of Psychiatry, Georgetown University School of Medicine, Washington, DC, Chairman, Department of Psychiatry, Suburban Hospital, a member of Johns Hopkins Medicine, Bethesda, MD

Issue
Current Psychiatry - 11(07)
Publications
Topics
Page Number
29-32
Legacy Keywords
suicide; suicide rehearsal; emergency; intervention
Sections
Author and Disclosure Information

 

Robert I. Simon, MD
Clinical Professor of Psychiatry, Georgetown University School of Medicine, Washington, DC, Chairman, Department of Psychiatry, Suburban Hospital, a member of Johns Hopkins Medicine, Bethesda, MD

Author and Disclosure Information

 

Robert I. Simon, MD
Clinical Professor of Psychiatry, Georgetown University School of Medicine, Washington, DC, Chairman, Department of Psychiatry, Suburban Hospital, a member of Johns Hopkins Medicine, Bethesda, MD

Article PDF
Article PDF

A suicide rehearsal is a behavioral enactment of a suicide method, usually as part of a suicide plan. A mental suicide rehearsal is a process that evolves over time into a plan. Patients who are intent on attempting suicide usually do not reveal their plans. However, behavioral rehearsals display specific clinical characteristics that speak louder than the guarded patient’s denials, revealing the patient’s suicide plan (Table).

Suicide rehearsals may precede suicide attempts or suicide completions. The percentage of patients who stage suicide rehearsals before attempting or completing suicide is unknown; however, in my experience, suicide rehearsals are relatively common. This article describes suicide rehearsals, and offers 4 cases that illustrate what clinicians can learn from rehearsals to improve their patients’ safety.

Table

Clinical characteristics of suicide rehearsals

 

Guarded patient
Behavioral enactment of a suicide method
Lethal means
Presumptive acute, high risk of suicide
Severe mental illness
Suicide attempt often within hours or days
Rehearsal usually covert
Rehearsal event or multiple events

The psychology behind suicide rehearsals

Rehearsing suicidal behavior can lower the barrier to a suicide plan, thereby increasing a patient’s resolve and risk. Joiner1 notes that engaging in behavioral or mental suicide rehearsals increases the risk of suicide. Moreover, rehearsals diminish the prohibition against suicidal behavior and the fear of pain and dying. Examples of rehearsal psychology include:

 

  • overcoming ambivalence about dying
  • desensitizing anxiety about performing the suicide act
  • testing or “perfecting” the method of a planned suicide
  • firming one’s resolve to complete suicide.

Other non-lethal motivations include “a cry for help” and self-injurious behaviors motivated by external gains. Patients who do not intend to attempt suicide may openly rehearse low-risk methods, such as superficial cutting.

Rehearsal characteristics

 

Suicide rehearsals can be confused with aborted, interrupted, or failed suicide attempts. Suicide rehearsals usually are associated with severe psychiatric illness and high-risk lethal methods of attempting suicide. My experience is that suicide attempts or suicide completions often follow a rehearsal within a few hours or days. However, no short-term suicide risk factors—within hours, days, or weeks—can predict when or if a rehearsed suicide will proceed to a suicide attempt.2

A suicide rehearsal is presumptive evidence that the patient is at acute, high risk for suicide and immediate clinical intervention is necessary. A rehearsal allows the clinician to explore the various methods of suicide that the patient has considered, including prior rehearsals. Knowledge of prior rehearsals can inform the clinician’s management of the current suicide rehearsal.

Suicide rehearsals often are conducted covertly. On inpatient psychiatric units, the rehearsal usually is discovered by staff members or reported by other patients. In outpatient settings, the patient or a significant other may report a rehearsal.

The suicide method displayed in a rehearsal may change. A patient who is rehearsing a hanging may attempt suicide by overdose or a firearm. In a systematic review of prior suicide attempts (N = 1,397), Isometsä et al3 found that 82% of patients used 2 or more different methods in suicide attempts, including the completed suicide. However, in a cohort study of 48,649 individuals admitted to a hospital after an attempted suicide, Runeson et al4 found that patients who attempt suicide often used the same method in completed suicide (ie, >90% by hanging for both men and women). Therefore, when taking measures to restrict the patient’s access to lethal means, safety efforts should not be limited to the method used in the suicide rehearsal. Patients can always substitute methods.

Making overall preparations for suicide—for example, making a will, giving away valuable possessions, or putting financial affairs in order—could be confused with a suicide rehearsal, which displays the lethal method to be used in a suicide attempt, often after preparations are made. Suicide rehearsals tend to occur much closer in time to the suicide attempt than preparations for suicide. Similarly, a patient’s plan to hoard drugs for a suicide attempt is not the same as ingesting a sub-lethal dose of a drug to test his or her resolve to die.

By definition, impulsive suicide attempts are not rehearsed. However, an individual’s suicide rehearsal can impulsively segue into a suicide attempt. In a case control study (N = 153) Simon et al5 found that 24% of patients spent 6 found that 26% of individuals with lifetime suicide ideation transitioned from suicide ideation to an unplanned suicide attempt. In my experience, a suicide rehearsal before a suicide completion is presumptive evidence against an impulsive suicide.

 

 

Patients contemplating suicide may visit Web sites with instructions on “how to suicide,” providing “virtual” opportunities to rehearse suicide.7 Patients who are at risk for suicide should be asked if they have searched the Internet for suicide methodology.

What we can learn from rehearsals

Although the following case examples are fictional, they illustrate suicide rehearsals encountered in my clinical and forensic practice.

CASE 1: Looking for a location

Ms. B, a 28-year-old divorced mother of 2, is observed tarrying at the high point of a bridge on successive days. When police arrive and question her, she becomes agitated and distraught. Ms. B admits to “scoping out” the bridge and is taken to a hospital emergency room (ER). In the ER, Ms. B discloses, “I was looking for a good spot to jump.” She tells the triage nurse that she is very depressed but, “I couldn’t do it to my children.” Ms. B is placed in an unlocked room while she waits to be assessed by a psychiatrist. She leaves the ER, runs to a nearby parking garage, and jumps from the top level to her death.

Comment: A patient’s denial of suicide intent following a suicide rehearsal cannot be relied upon. Ms. B’s rehearsal revealed a plan with high-risk suicide intent and a lethal suicide method. Systematic suicide assessment that informs immediate clinical intervention is required.

CASE 2: Changing lethal means

Mr. N, a 43-year-old chief executive officer of a large company, is observed by an assistant loading and unloading a revolver at his desk. Alarmed, the assistant calls the company physician. Mr. N refuses psychiatric treatment, saying, “I’ll be all right; this is just a passing thing.” His wife tells the physician that her husband has a history of bipolar disorder but no prior suicide attempts. Guns and ammunition are removed from the home. One week later, Mr. N is found hanging in his garage. A loaded pistol is discovered in the glove compartment of his car.

Comment: There is no certainty that a subsequent suicide attempt will replicate the rehearsed method. A psychological autopsy was conducted, but no explanation was found for why Mr. N chose hanging after having rehearsed suicide with a loaded handgun. His wife thought that her husband, a very tidy person, did not want to leave a mess.

CASE 3: Grieving and depressed

 

Mr. O, age 67, is depressed after recently losing his wife. He considers a number of suicide methods. Mr. O decides to use a plastic bag to suffocate himself because he believes that this method will allow him to change his mind. Mr. O practices tying the bag tight around his neck. During this rehearsal, he realizes that he does not want to die. Instead, he pursues grief counseling.

 

Comment: For some patients, the act of rehearsing suicide can help them resolve ambivalent feelings about wanting to die in favor of wanting to live.

 

CASE 4: Suicide method and the Internet

Ms. S, a 22-year-old college student, is undergoing outpatient treatment for depression. She is accumulating prescription drugs to take as an overdose. Ms. S also searches the Internet for information about other suicide methods. Because she wants a “sure” method of suicide, she persuades an acquaintance to purchase a handgun. In private, Ms. S places the unloaded gun to her head and plays “Russian roulette,” pulling the trigger several times. Her mother discovers the gun and confronts her daughter. Ms. S is hospitalized on a closed psychiatric unit and tells a staff member, “I was practicing suicide with the gun.” Before Ms. S is discharged from the hospital, her parents are advised to watch for suicidal behaviors, especially the recurrence of rehearsals that indicate an acute, high suicide risk. Ms. S’s Internet use is restricted and monitored.

Comment: Suicide rehearsal with a gun reinforces the belief that a firearm death is quick and easy.8 Reaching for a loaded gun takes less time than most other methods of suicide. Patients who rehearse suicide with a gun should be prevented from having access to any firearms, weapons, or other highly lethal means of suicide.

Recognition and intervention

A guarded psychiatric inpatient who is intent on attempting suicide is unmasked when the discovery of a suicidal rehearsal reveals a suicide plan. This creates an opportunity for clinicians to intervene. The patient may attempt to cover up suicidal intent by stating, “I was just playing around” or “I just wanted to get attention.” Recognizing the emergency posed by a suicide rehearsal informs treatment. Safety measures—including 1-to-1 supervision—may be necessary during a period of acute, high suicide risk. The patient’s diagnosis, severity of illness, and treatment require reevaluation.

 

 

 

An outpatient who performs a suicide rehearsal should be considered at acute, high risk for suicide, and immediate psychiatric hospitalization may be necessary. Whether as an inpatient or outpatient, the patient’s suicide intent and plan require careful exploration. The information gained will guide treatment and management decisions. Continuing systematic suicide risk assessment is essential.

Related Resources

 

  • American Association of Suicidology. www.suicidology.org.
  • Joiner T. Why people die by suicide. Cambridge, MA: Harvard University Press; 2007.
  • American Psychiatric Association. Practice guideline for the assessment and treatment of patients with suicidal behaviors. Washington, DC: American Psychiatric Publishing, Inc. 2003.
  • Simon RI. Preventing patient suicide: clinical assessment and management. Arlington, VA: American Psychiatric Publishing, Inc.; 2011.

Disclosure

Dr. Simon reports no financial relationship with any company whose products are mentioned in this article or with manufacturers of competing products.

A suicide rehearsal is a behavioral enactment of a suicide method, usually as part of a suicide plan. A mental suicide rehearsal is a process that evolves over time into a plan. Patients who are intent on attempting suicide usually do not reveal their plans. However, behavioral rehearsals display specific clinical characteristics that speak louder than the guarded patient’s denials, revealing the patient’s suicide plan (Table).

Suicide rehearsals may precede suicide attempts or suicide completions. The percentage of patients who stage suicide rehearsals before attempting or completing suicide is unknown; however, in my experience, suicide rehearsals are relatively common. This article describes suicide rehearsals, and offers 4 cases that illustrate what clinicians can learn from rehearsals to improve their patients’ safety.

Table

Clinical characteristics of suicide rehearsals

 

Guarded patient
Behavioral enactment of a suicide method
Lethal means
Presumptive acute, high risk of suicide
Severe mental illness
Suicide attempt often within hours or days
Rehearsal usually covert
Rehearsal event or multiple events

The psychology behind suicide rehearsals

Rehearsing suicidal behavior can lower the barrier to a suicide plan, thereby increasing a patient’s resolve and risk. Joiner1 notes that engaging in behavioral or mental suicide rehearsals increases the risk of suicide. Moreover, rehearsals diminish the prohibition against suicidal behavior and the fear of pain and dying. Examples of rehearsal psychology include:

 

  • overcoming ambivalence about dying
  • desensitizing anxiety about performing the suicide act
  • testing or “perfecting” the method of a planned suicide
  • firming one’s resolve to complete suicide.

Other non-lethal motivations include “a cry for help” and self-injurious behaviors motivated by external gains. Patients who do not intend to attempt suicide may openly rehearse low-risk methods, such as superficial cutting.

Rehearsal characteristics

 

Suicide rehearsals can be confused with aborted, interrupted, or failed suicide attempts. Suicide rehearsals usually are associated with severe psychiatric illness and high-risk lethal methods of attempting suicide. My experience is that suicide attempts or suicide completions often follow a rehearsal within a few hours or days. However, no short-term suicide risk factors—within hours, days, or weeks—can predict when or if a rehearsed suicide will proceed to a suicide attempt.2

A suicide rehearsal is presumptive evidence that the patient is at acute, high risk for suicide and immediate clinical intervention is necessary. A rehearsal allows the clinician to explore the various methods of suicide that the patient has considered, including prior rehearsals. Knowledge of prior rehearsals can inform the clinician’s management of the current suicide rehearsal.

Suicide rehearsals often are conducted covertly. On inpatient psychiatric units, the rehearsal usually is discovered by staff members or reported by other patients. In outpatient settings, the patient or a significant other may report a rehearsal.

The suicide method displayed in a rehearsal may change. A patient who is rehearsing a hanging may attempt suicide by overdose or a firearm. In a systematic review of prior suicide attempts (N = 1,397), Isometsä et al3 found that 82% of patients used 2 or more different methods in suicide attempts, including the completed suicide. However, in a cohort study of 48,649 individuals admitted to a hospital after an attempted suicide, Runeson et al4 found that patients who attempt suicide often used the same method in completed suicide (ie, >90% by hanging for both men and women). Therefore, when taking measures to restrict the patient’s access to lethal means, safety efforts should not be limited to the method used in the suicide rehearsal. Patients can always substitute methods.

Making overall preparations for suicide—for example, making a will, giving away valuable possessions, or putting financial affairs in order—could be confused with a suicide rehearsal, which displays the lethal method to be used in a suicide attempt, often after preparations are made. Suicide rehearsals tend to occur much closer in time to the suicide attempt than preparations for suicide. Similarly, a patient’s plan to hoard drugs for a suicide attempt is not the same as ingesting a sub-lethal dose of a drug to test his or her resolve to die.

By definition, impulsive suicide attempts are not rehearsed. However, an individual’s suicide rehearsal can impulsively segue into a suicide attempt. In a case control study (N = 153) Simon et al5 found that 24% of patients spent 6 found that 26% of individuals with lifetime suicide ideation transitioned from suicide ideation to an unplanned suicide attempt. In my experience, a suicide rehearsal before a suicide completion is presumptive evidence against an impulsive suicide.

 

 

Patients contemplating suicide may visit Web sites with instructions on “how to suicide,” providing “virtual” opportunities to rehearse suicide.7 Patients who are at risk for suicide should be asked if they have searched the Internet for suicide methodology.

What we can learn from rehearsals

Although the following case examples are fictional, they illustrate suicide rehearsals encountered in my clinical and forensic practice.

CASE 1: Looking for a location

Ms. B, a 28-year-old divorced mother of 2, is observed tarrying at the high point of a bridge on successive days. When police arrive and question her, she becomes agitated and distraught. Ms. B admits to “scoping out” the bridge and is taken to a hospital emergency room (ER). In the ER, Ms. B discloses, “I was looking for a good spot to jump.” She tells the triage nurse that she is very depressed but, “I couldn’t do it to my children.” Ms. B is placed in an unlocked room while she waits to be assessed by a psychiatrist. She leaves the ER, runs to a nearby parking garage, and jumps from the top level to her death.

Comment: A patient’s denial of suicide intent following a suicide rehearsal cannot be relied upon. Ms. B’s rehearsal revealed a plan with high-risk suicide intent and a lethal suicide method. Systematic suicide assessment that informs immediate clinical intervention is required.

CASE 2: Changing lethal means

Mr. N, a 43-year-old chief executive officer of a large company, is observed by an assistant loading and unloading a revolver at his desk. Alarmed, the assistant calls the company physician. Mr. N refuses psychiatric treatment, saying, “I’ll be all right; this is just a passing thing.” His wife tells the physician that her husband has a history of bipolar disorder but no prior suicide attempts. Guns and ammunition are removed from the home. One week later, Mr. N is found hanging in his garage. A loaded pistol is discovered in the glove compartment of his car.

Comment: There is no certainty that a subsequent suicide attempt will replicate the rehearsed method. A psychological autopsy was conducted, but no explanation was found for why Mr. N chose hanging after having rehearsed suicide with a loaded handgun. His wife thought that her husband, a very tidy person, did not want to leave a mess.

CASE 3: Grieving and depressed

 

Mr. O, age 67, is depressed after recently losing his wife. He considers a number of suicide methods. Mr. O decides to use a plastic bag to suffocate himself because he believes that this method will allow him to change his mind. Mr. O practices tying the bag tight around his neck. During this rehearsal, he realizes that he does not want to die. Instead, he pursues grief counseling.

 

Comment: For some patients, the act of rehearsing suicide can help them resolve ambivalent feelings about wanting to die in favor of wanting to live.

 

CASE 4: Suicide method and the Internet

Ms. S, a 22-year-old college student, is undergoing outpatient treatment for depression. She is accumulating prescription drugs to take as an overdose. Ms. S also searches the Internet for information about other suicide methods. Because she wants a “sure” method of suicide, she persuades an acquaintance to purchase a handgun. In private, Ms. S places the unloaded gun to her head and plays “Russian roulette,” pulling the trigger several times. Her mother discovers the gun and confronts her daughter. Ms. S is hospitalized on a closed psychiatric unit and tells a staff member, “I was practicing suicide with the gun.” Before Ms. S is discharged from the hospital, her parents are advised to watch for suicidal behaviors, especially the recurrence of rehearsals that indicate an acute, high suicide risk. Ms. S’s Internet use is restricted and monitored.

Comment: Suicide rehearsal with a gun reinforces the belief that a firearm death is quick and easy.8 Reaching for a loaded gun takes less time than most other methods of suicide. Patients who rehearse suicide with a gun should be prevented from having access to any firearms, weapons, or other highly lethal means of suicide.

Recognition and intervention

A guarded psychiatric inpatient who is intent on attempting suicide is unmasked when the discovery of a suicidal rehearsal reveals a suicide plan. This creates an opportunity for clinicians to intervene. The patient may attempt to cover up suicidal intent by stating, “I was just playing around” or “I just wanted to get attention.” Recognizing the emergency posed by a suicide rehearsal informs treatment. Safety measures—including 1-to-1 supervision—may be necessary during a period of acute, high suicide risk. The patient’s diagnosis, severity of illness, and treatment require reevaluation.

 

 

 

An outpatient who performs a suicide rehearsal should be considered at acute, high risk for suicide, and immediate psychiatric hospitalization may be necessary. Whether as an inpatient or outpatient, the patient’s suicide intent and plan require careful exploration. The information gained will guide treatment and management decisions. Continuing systematic suicide risk assessment is essential.

Related Resources

 

  • American Association of Suicidology. www.suicidology.org.
  • Joiner T. Why people die by suicide. Cambridge, MA: Harvard University Press; 2007.
  • American Psychiatric Association. Practice guideline for the assessment and treatment of patients with suicidal behaviors. Washington, DC: American Psychiatric Publishing, Inc. 2003.
  • Simon RI. Preventing patient suicide: clinical assessment and management. Arlington, VA: American Psychiatric Publishing, Inc.; 2011.

Disclosure

Dr. Simon reports no financial relationship with any company whose products are mentioned in this article or with manufacturers of competing products.

References

 

1. Joiner TE, Jr. The trajectory of suicidal behavior over time. Suicide Life Threat Behav. 2002;32(1):33-41.

2. Simon RI. Imminent suicide: the illusion of short-term prediction. Suicide Life Threat Behav. 2006;36(3):296-301.

3. Isometsä ET, Lönnqvist JK. Suicide attempts preceding completed suicide. Br J Psychiatry. 1998;173:531-535.

4. Runeson B, Tidemalm D, Dahlin M, et al. Method of attempted suicide as a predictor of subsequent successful suicide: national long-term cohort study. BMJ. 2010;341:c3222. doi:10.1136/bmj.63222.

5. Simon TR, Swann AC, Powell KE, et al. Characteristics of impulsive suicide attempts and attempters. Suicide Life Threat Behav. 2001;32(suppl):49-59.

6. Kessler RC, Borges G, Walters EE. Prevalence of and risk factors for lifetime suicide attempts in the National Comorbidity Study. Arch Gen Psychiatry. 1999;56(7):617-626.

7. Recupero PR. Suicide and the Internet. In: Simon RI Hales RE, eds. The American Psychiatric Publishing textbook of suicide assessment and management. 2nd ed. Arlington, VA: American Psychiatric Publishing, Inc.; 2012:515–538.

8. Simon RI. Gun safety management with patients at risk for suicide. Suicide Life Threat Behav. 2007;37(5):518-526.

References

 

1. Joiner TE, Jr. The trajectory of suicidal behavior over time. Suicide Life Threat Behav. 2002;32(1):33-41.

2. Simon RI. Imminent suicide: the illusion of short-term prediction. Suicide Life Threat Behav. 2006;36(3):296-301.

3. Isometsä ET, Lönnqvist JK. Suicide attempts preceding completed suicide. Br J Psychiatry. 1998;173:531-535.

4. Runeson B, Tidemalm D, Dahlin M, et al. Method of attempted suicide as a predictor of subsequent successful suicide: national long-term cohort study. BMJ. 2010;341:c3222. doi:10.1136/bmj.63222.

5. Simon TR, Swann AC, Powell KE, et al. Characteristics of impulsive suicide attempts and attempters. Suicide Life Threat Behav. 2001;32(suppl):49-59.

6. Kessler RC, Borges G, Walters EE. Prevalence of and risk factors for lifetime suicide attempts in the National Comorbidity Study. Arch Gen Psychiatry. 1999;56(7):617-626.

7. Recupero PR. Suicide and the Internet. In: Simon RI Hales RE, eds. The American Psychiatric Publishing textbook of suicide assessment and management. 2nd ed. Arlington, VA: American Psychiatric Publishing, Inc.; 2012:515–538.

8. Simon RI. Gun safety management with patients at risk for suicide. Suicide Life Threat Behav. 2007;37(5):518-526.

Issue
Current Psychiatry - 11(07)
Issue
Current Psychiatry - 11(07)
Page Number
29-32
Page Number
29-32
Publications
Publications
Topics
Article Type
Display Headline
Suicide rehearsals: A high-risk psychiatric emergency
Display Headline
Suicide rehearsals: A high-risk psychiatric emergency
Legacy Keywords
suicide; suicide rehearsal; emergency; intervention
Legacy Keywords
suicide; suicide rehearsal; emergency; intervention
Sections
PURLs Copyright

Disallow All Ads
Alternative CME
Use ProPublica
Hide sidebar & use full width
render the right sidebar.
Conference Recap Checkbox
Not Conference Recap
Clinical Edge
Display the Slideshow in this Article
Article PDF Media